Вы находитесь на странице: 1из 128

GENETICS ​FINAL​ QUESTION COLLECTOR  

 
 
Downloading, sharing and adding people to these docs is strictly forbidden. 
This will not be tolerated and you shall be eliminated from all the docs, even if it’s Mohit. 
FORMAT RULES 

● Font: CAIRO 
● Size: 11 
● Spacing: 1.5 
● Question in bold 
 

 
KEY 
Answer ​= Dipika, Inder, Ritika, etc answer[TEMPORARY] 
Answer​ = MOHIT’s, EMILIA answer [PERMANENT] 
like this​ (something ​ in blue) ​=​ The correct information 
 
 
CONTENTS: 

2018 FINAL (2nd Year)​ ​= SOLVED  2 

2017 FINAL​ ​= SOLVED  10 

POLISH DIVISION 2018 FINAL​ ​= SOLVED  20 

2016 FINAL​ ​= SOLVED  29 

2016 FINAL Retake​ ​= SOLVED  40 

MCQ-1 48 

2015 4yr 53 

POLISH DIVISION 2017 57 

RANDOM 1 69 

RANDOM 2 71 
MCQ’s FROM BOOK​ ​= SOLVED  72 

2009 FINAL 94 

MCQ-2 104 

RANDOM YEAR 114 


 

2018 FINAL (2nd Year)  


1. Choose the correct set of answers (T-true, F-false) for the statements: 
1)  For  estimating  the  recurrence  risk  for  siblings  of  a  person  with  the  disease  caused  by  de  novo 
mutation a Punnett square should be used 
2) De novo mutation cannot occur in genes which are linked  
3) Germline mosaicism cannot be responsible for recurrence of Duchenne muscular dystrophy 
4) De novo mutations always have phenotypic effect 
5) De novo mutations cannot occur in genes linked to polygenic disease 
A. 1F, 2T, 3F, 4F, 5T 
B. 1F, 2T, 3T, 4T,5F 
C. 1F, 2F, 3F, 4F, 5F 
D. 1T, 2T, 3F, 4T, 5F 
 
2) In this pedigree:  
A. Both parents are carriers 
B. The disease most probably occurred as a result of de novo mutation 
C. X-linked or autosomal recessive model of inheritance is likely 
D. Answer A and C are correct 
 
3)  In  autosomal  dominant  disease  the  normal  allele  in  population has the 
frequency  of  0.996.  The  disease  is  not  genetically  lethal,  but  it  affected  people  have  an 
approximately 1/4 fewer children. The frequency of the mutation causing this disease equals: 
A. 0.0005 
B. 0.001 
C. 0.1245 
D. 0.001 
 
4) Choose: 
A. In  autosomal  dominant  diseases,  the  mutated  allele  frequency  equals  the  frequency  of  the 
disease divided by two 
B. In  autosomal  dominant disease, the frequency of the disease equals a half of the mutated allele 
frequency 
C. In  X-linked  disease  the  frequency  of  the  affected  men  equals  a  half  of  the  frequency  of  the 
mutated allele 
D. Answer is B and C are correct 
 
5) Which point mutations ​cannot​ be detected by PCR and Sanger sequencing 
A. deletion of five base pairs tag 
B. Duplication of 100 kip 
C. Insertion of thymine 
D. Substitution of adenine into guanine 
 
6) Select the correct answer for the ​Sanger sequencing​: 
A. Detects deletions and duplications above 10 kb 
B. It Is an alternative method to FISH 
C. Allows for the Analysis of the whole genome in one experiment 
D. Enable the detection of point mutation and analyzed fragment of DNA 
 
7) The unit of genetic mapping is 1 CentImorgan (1cM) that corresponds to: 
A. 1 kilo base pair distance between two genes 
B. 1% probability of linkage 
C. Lod score amounting to 1 
D. 1% frequency of crossing-over between two genes 
 
8) Mark corrects about linkage analysis 
A. In  a  3  generation  family,  the phase is usually unknown and the calculations are carried out for 2 
phases 
B. In a 2 generation family, that phase is unknown 
C. In a 2 generation family, the phase is known and calculations are carried out for two phases 
D. In two families with the same disease, the phases are usually consistent 
 
9)  Indicate  the  correct  answer  for  the  linkage  Analysis  assuming  that  the  disease  is  inherited  in  a 
autosomal  dominant  pattern  and  in  the  adopted  phase  the  disease Segregated with the 4 allele. NR 
is the person who hasn’t recombined; R is the person who has recombined. 
A. NR, R, NR, NR 
B. R, R, R, R 
C. R, NR, R, R 
D. NR, NR, NR, NR 

 
 
10) Select the right answer: 
A. “The gold standard”, used for pathogenicity estimation in prospective studies, is odds ratio ​(OR) 
B. “The  gold standard”, used for pathogenicity estimation in prospective studies, is relative risk 
(RR) 
C. “The  gold  standard”,  used  for  pathogenicity  estimation  in  retrospective  studies,  is  relative  risk 
(RR) n 
D. It is possible to calculate relative risk (RR) in retrospective studies 
 

  Disease  Disease 
+  - 

Mutation +  50  1000 

Mutation -  25  500 

 
11) When is the value of odds ratio (OR) similar to the value of risk ratio (RR) 
a. When  the  probability  of  getting  the  illness  in  the  study  group  is  high,  while  the  probability  for 
the control group is low 
b. When  the probability of getting the illness in the study group is low, while the probability for the 
control group is high 
c. When the probability of getting the illness for both groups is low 
d. Never, the values of OR and RR always differ 
 
12) What is the value of odds ratio (OR) in the retrospective study above? 
a. 1  
b. 0,1 
c. 10 
d. 100 
 
13) Is the prospective study carried out properly? 

  Disease  Disease 
+  - 

Mutation +  5  195 

Mutation -  3  197 

 
a. Yes, because both the study group and the control group are equally large  
b. No, because both the study group and the control group are equally large  
c. Yes, because in the study group and the control group the number of affected patients is low 
d. No, because in the study group and the control group the number of affected patients is low  
 
14)  Autosomal  recessive  inheritance.  Parents  are  carriers.  The  risk  of  healthy  child  being  a  carrier 
equals: 
a. 25% 
b. 33% 
c. 50% 
d. 66% 
 
15)  Autosomal  recessive  inheritance.  The  status  of  parents  carrier  is  unknown.  The  disease 
frequency in the population is 0.0004. The approximate risk of being a carrier for each parent is: 
a) 0.005 
b) 0.002 
c) 0.04 
d) 0.02 
 
16)  X-linked  inheritance.  A  healthy  woman  has  a  sick  brother,  two  healthy  sons  and  a  healthy 
daughter. Grandparents (her parents) are healthy. The risk of being a carrier for a daughter is: 
a) There is no risk for being a carrier 
b) 1% 
c) 10% 
d) 50% 
 
17)  Autosomal  dominant  inheritance.  One  of  the  parents  is  affected  heterozygote,  the  other  is  a 
healthy  homozygote.  If  the  penetrance  is  100%, the probability of having a healthy child is 50%. How 
will the probability of having a healthy child change if the penetrance equals 80%?: 
a) Will not change (50%) 
b) Will increase (60%) 
c) Will increase (80%) 
d) Will decrease (20%) 
 
18) Which karyotype designates the male patient with ​Patau syndrome​?  
a) 47,XY,​+13   
b) 45,XY,der(13;21) (p11;p11)​+13 
c) 47,XY ​+13​[20]/46,XY[20] 
d) All answers are correct 
 
19) Indicate an ​incorrect​ statement: 
a) Array  CGH  offers  karyotype  analysis  with  no  need  of  cell  culturing,  high  resolution  and 
enables to detection of all type of chromosomal aberrations  
b) G-banding  of  chromosomes  during  prometaphase  provide  higher  resolution  than  during 
metaphase 
c) Interphase  FISH  is  commonly  used  in  the  analysis  of  chromosomal  rearrangements  in  tumor 
cells 
d) Spectral karyotype is especially useful for the identification of chromosomal translocations.  
 
20) Indicate a parent with the highest risk of having child with Down syndrome: 
a) 25-year old woman with previous Down- syndrome child  
b) 25-year old female carrier of a chromosomal aberration 45,XX,der(14;21) (p11;p11) 
c) 25-year old male carrier of chromosomal aberration 45,XY,der(14;21) (p11;p11) 
d) 45-year  old  woman  with  no  previous  history  of  Down  syndrome  child  (mother  of  two  healthy 
children)  
 
21) Indicate a balanced chromosomal aberration: 
a) Reciprocal translocation 
b) Terminal deletion 
c) Isochromosome  
d) All answers are correct 
 
22) Approximately 70% of cases of ​Prader-Willi syndrome ​is caused by: 
a) Uniparental disomy of chromosome 13 inherited from father 
b) Duplication of 13q11-13 region on the maternally inherited chromosom​e  
c) Deletion of 15q11-13 region on paternally inherited chromosome 
d) Deletion of 15p11-13 region on paternally inherited chromosome  
 
23) Indicate the ​incorrect​ sentence describing X-linked recessive pattern of inheritance:  
a) Females carriers are asymptomatic  
b) Hemizygous male transfer mutated allele to all sons, never to daughters 
c) Females transmit mutated alleles to sons as well as to daughters 
d) All daughters of an affected male are mandatory carriers 
 
24. Indicate the ​incorrect​ sentence describing genomic imprinting: 
a) It’s  a  specific  kind  of  epigenetic  mechanism  of  gene 
expression 
b) One of the mechanisms of genomic imprinting is methylation of inactive allele 
c) This sex-specific genomic modification occurs during conception 
d) Inactive  alleles  are  inherited  according  to  Mendel’s  rules,  whereas  expression  depends  on  the 
sex of a parent from whom this allele came. 
 
25.  Indicate  appropriate  pedigree  for  family  with  autosomal  dominant  disease  which  comes  under 
paternal genomic imprinting (meaning the paternally inherited allele is inactive). ​c) 

2017 FINAL 
1. The  diagnosis  of  a  genetic  cause  of  autism  spectrum  disorder  is  MOST  likely  in  an 
individual with: 
A. Essential autism 
B. Autism + microcephaly 
C. Autism + congenital heart disease 
D. Autism + extrapyramidal disorder + dysmorphism  
  
2. In autosomal recessive conditions: 
A. The healthy sibling of an affected person bears a 2 in 3 risk of being a carrier 
B. The risk of the affected offspring if both parents are carriers is 50%  
C. Parental consanguinity does not increase the risk of such condition of the offspring 
D. All above 
 
3. If  the  population  frequency  of  an  autosomal  recessive  disease  is  1/2500,  then, 
according  to  the  Hardy-Weinberg  equilibrium,  the  frequency  of  the  mutated  allele 
and the frequency of carriers, respectively, are approximately: 
A. 1/50 and 1/25 
B. 1/25 and 1/50 
C. 1/1,250 and 1/25 
D. 1/25 and 1/1,250 
 
4. A  typical  clinical  indication  for  ​Preimplantation  Genetic  Diagnosis  (PGD)  is 
establishing: 
A. A deletion of SMN1 gene in both mates (autosomal recessive spinal muscular atrophy) 
B. A pathogenic variant in CFTR gene in one mate (autosomal recessive cystic fibrosis) 
C. A  history  of  autosomal  dominant  osteogenesis  imperfecta in one mates family without 
genetic testing ever performed 
D. All above 
 
5. Indicate the true statement for ​turner syndrome​: 
A. Majority of affected individuals have intellectual disability​
B. Secondary sex characteristics in male 
C. Great majority of cases are sporadic 
D. None of the above is true 
 
6. In the pedigree below, indicate who should be genetically tested first: 
A. The consultant 
B. Any one of the living affected 
C. Choice of the one to be tested doesn't change the odds of detecting pathogenic variant 
D. No one, as this is not a pattern characteristics of hereditary cancer 

 
  
7. Early onset breast cancer and soft tissue sarcomas are the hallmark of: 
A. Cowden syndrome 
B. Lynch syndrome 
C. Peutz jeghers syndrome 
D. Li-fraumeni syndrome 
 
8. Increased  nuchal  translucency  (NT)  above  two  standard  deviations  (>250)  at  11-13 
wks of pregnancy ​points to a possible: 
A. Down syndrome 
B. isolated congenital heart defect 
C. normal outcome of pregnancy 
D. all of the above 
 
9. The so-called ​chromosomal phenotype​ includes the following ​except​: 
A. mental retardation 
B. facial dysmorphism 
C. isolated congenital heart defect 
D. multiple congenital anomalies 
 
10. Which of the following syndromes could​ not ​be diagnosed by classic karyotyping: 
A. Down syndrome 
B. Klinefelter syndrome 
C. neurofibromatosis type 1 
D. Edwards syndrome 
 
11. Which following statements are true for prader-willi syndrome ​except 
A. it is one of the most commonly recognized genetic forms of obesity 
B. it  manifests  with  Central  hypotonia  and  feeding  difficulties  with  failure  to  thrive  in 
infancy 
C. it can be caused by a microdeletion on the ​maternal​ chromosome 15 
D. hypogonadism is part of the clinical picture 
 
12. Spinal muscular atrophy is caused by 
A. smn2 gene mutation 
B. smn1 Gene point mutation 
C. deletion of the Exon 7 of smn1 
D. recessive definition of Exon 7 of smn1 
 
13. Exon skipping technology: 
A. Is not used in neuromuscular disorders 
B. Can be used for point mutations only 
C. Results in DMD gene duplication 
D. Can be used for Duchenne muscular dystrophy  
 
14. Charcot-Marie-Tooth (CMT) disease type 1X: 
A. Is the most common form of CTM 
B. Is inherited from male to male 
C. Is inherited from male to male with an early onset disease present in males  
D. Is inherited as an X-linked dominant trait  
 
15. TP53, RB1, APC, BRCA1, BRCA2 are examples of: 
A. Tumor suppressor genes 
B. (Proto)oncogenes 
C. Transcription factors 
D. Tyrosine kinase inhibitors  
 
16. Explain the following: 46, XY, der (19),t(4;19)(p15.3;q13.2)mat 
A. A  male  with  trisomy  of  a fragment of chromosome 4 short arm and a monosomy of a 
fragment of chromosome 19 long arm  
B. A  male  with  a  trisomy  of  a  fragment  of  chromosome 19 short arm and a monosomy of 
a fragment of chromosome 4 long arm  
C. A  male  with  a  trisomy of a fragment of chromosome 19 long arm and a monosomy of a 
terminal fragment of chromosome 4 short arm 
D. A  male  with  a  trisomy  of  a  fragment  of  chromosome  4  long  arm  and a monosomy of a 
fragment of chromosome 19 short arm 
 
17. Lynch syndrome refers to inherited susceptibility of: 
A. Skin cancer (melanoma) 
B. Colorectal cancer (CRC) 
C. Small cell lung cancer (SCLC) 
D. Non-small cell lung cancer (NSCLC) 
 
18. Gene  mutation  associated  with  polyprotein structures alteration that can disrupt the 
activity if the wild-type gene when overexpressed is called: 
A. Haploinsufficiency 
B. Two hit theory 
C. Clonal theory of cancer 
D. The dominant negative effect 
 
19. Breast  cancer  caused  by  BRCA2  mutation  is  available  for  treatment  with  PARP 
inhibitors (poly-ADP-ribose polymerase) by exploiting: 
A. Crossing-over 
B. Dominant negative effect 
C. Negative feedback 
D. Synthetic lethality 
 
20. ​A ​pleiotropic gene​ is the one which: 
A. Is expressed in one specific tissue only 
B. Is present in the genome in multiple copies 
C. Affects more than one trait 
D. Due to its size, it is especially exposed to mutations 
 
21. Indicate karyotype for an unbalanced chromosomal aberration 
A. 46, XX, del(13)(q12;q13) 
B. 46, XY, inv(5)(q20;q22) 
C. 45, XX, der(13;14)(q10;q10) 
D. A and C 
 
22. Assuming  a  dominant  X-linked  inheritance  pattern,  penetrance  of  100%  and 
frequency  of  mutated  allele  of  2x10^-1,  the  likelihood  of  the  pedigree  (the  disease 
occurrence in a family of four as on the scheme) is approximately: 
 
 
A. 0,032 
B. 0,064 
C. 0,128 
D. 0,2 
 
23. Indicate a true statement: 
A. Heteroplasmy is the presence of both mutated and wild-type DNA in the cell nucleus 
B. Depletion  of  mitochondrial  DNA  (reduction  in  the  number  of  mtDNA  copies  in  the 
cell) is a cause of certain diseases with the dysfunction of mitochondria 
C. Crossover  between  nuclear  and  mitochondrial  DNA  enables  the  transmission  of 
mitochondrial traits from father 
D. All of the above are false 
 
 
24. What are the odds that the tested marker is fully linked (theta=0) to a causative gene 
for  an  autosomal  dominant  disease  in  the  family  showed  on  the  picture.  Results  of 
the genetic marker genotyping are indicated on the pedigree. 
 
 
 
 
 
 
 
 
 
 
A. 1 
B. 16 
C. 32 
D. 64 
 
25. An  individual  has  a  mutation  causing  a  disease (e.g. retinoblastoma), yet they do not 
develop  any  symptoms  in  their  lifetime.  Which  mechanism  is  responsible  for  such 
situation? 
A. Reduced penetrance 
B. Varied expression 
C. Germinal mutation  
D. Locus heterogeneity  
 
26. The  analysis  of  metaphasic  chromosomes  after  G-band  staining  ​DOES  NOT  give 
possibility of: 
A. Evaluating complete karyotype in a single test 
B. Detecting both numerical and structural aberrations  
C. Detecting microdeletions 
D. Detecting Robertsonian translocations 
 
27. The most probable inheritance pattern for the pedigree below is: 
 
 
 
 
 
 
A. Autosomal dominant 
B. Autosomal recessive 
C. X-linked recessive  
D. X-linked dominant  
 
28. Indicate a true statement. Genetically lethal diseases….: 
A. Always cause death of an individual 
B. Have the selection coefficient equal to 0 
C. Have a frequency which, among others, depends on the mutation rate 
D. Always result from a de novo mutation 
 
29. Two  days  ago,  a  25  year  old  woman gave birth to a healthy child. Her mother (child’s 
grandmother)  ​in  her  childhood  had  a  surgery  due  to  retinoblastoma.  The  young 
woman  herself  has  never  suffered  from  this  cancer  but  she wants to know what the 
probability  of  the  disease  is  in  her  newborn.  The  child’s  father and grandfather both 
are wild-type homozygotes. To determine genetic risk, you will take into account: 
A. Disease frequency in population 
B. Lack of disease in mother 
C. Lack of disease in child 
D. Both B and C 
 
30. Which of the statements about aCGH is true? 
A. Is is the cheapest type of a cytogenetic test 
B. It does not detect balanced translocations 
C. Its resolution is the same as for FISH method 
D. All are false 
 
31. Which  pedigree  is  the  most  probable  (taking  into  account  the  number  and  sex  of 
affected  individuals  in  I  and  II  generation)  for  autosomal  dominant  inheritance  with 
paternal​ imprinting (paternal allele is inactivated)? 

 
A. Pedigree A 
B. Pedigree B 
C. Pedigree C 
D. Pedigree D 
 
32. Next generation sequencing (NGS) is: 
A. A sequencing based on capillary electrophoresis 
B. A sequencing based on capillary electrophoresis with the use of fluorescent terminators 
C. A group of novel DNA sequencing techniques based mostly on microarrays 
D. None above 
 
33. The  frequency  of  AA genotype among affected individuals is 50%, and among healthy 
individuals  -  20%.  The  Odds  Ratio  (OR)  for  the  occurence  of  the  disease  associated 
with AA genotype is 
A. 2 
B. 2,5 
C. 3 
D. 4 
 
34. Which of the statements about mutation rate in germ cells is true? 
A. Chromosomal mutations are more frequent in females 
B. Point mutations are more frequent in males 
C. In both sexes, mutations occur in germ cells more frequently with age 
D. All statements are true  
 
35. A  4  year  old  child  with  mild  mental  retardation,  congenital  heart  defect: 
supravalvular  aortic  stenosis​,  joint  laxity,  facial  dysmorphism:  periorbital  fullness, 
wide mouth, full lips, and ​over-friendly personality​ presents typical symptoms of: 
A. DiGeorge syndrome 
B. Williams syndrome 
C. Angelman syndrome 
D. Prader-Willi syndrome  

POLISH DIVISION 2018 FINAL  


1. Choose the genes responsible for the transformation of a hypothetical gonad into a testicle: 
a) SRY, SOX9, SF1 
b) WNT4, SRY, SF1 
c) SRY, SOX9, DAX1 
d) SRY, DAX1, WNT4 
 
2. Which of these karyotypes is not possible in Turner’s syndrome? 
a) 45,X 
b) 45,X/46,XX 
c) 46,X,i(Xq) 
d) All of these karyotypes are possible 
 
3. Ovotestis is​: 
a) The presence of both gonads: testicles and ovaries in one person 
b) A gonad containing both testicular and ovarian aspects 
c) True hermaphroditism 
d) Prader stage V genitalia 
 
4. Choose the correct karyotype of a girl with Down’s syndrome: 
a) 47,XX, +21 
b) 46,XX,der(14;21)(q10;q10),+21 
c) 46,XX [10] / 47,XX,+21 [40] 
d) All answers are correct 
 
5. Choose the karyotype of a Robertsonian Translocation: 
a) 46,XY ,t(15,22)(q10;q10) 
b) 45,XY ,der(13,22)(q10;q10) 
c) 45,XY ,t(14;20)(q10;q10) 
d) 46,XY ,der(13;21)(q10;q10) 
 
6. Choose the correct sentence: 
a) Numerical chromosomal aberrations in autosomal chromosomes are always lethal 
b) Reciprocal translocations are ​un​balanced 
c) Paracentric inversions engages p and q arms of the chromosome and the centromere 
d) A  ring  chromosome  forms  as  a  result  of  terminal  deletions  of  chromosomal  arms  and  their 
subsequent fusion 
 
7. Choose the genogram in which woman II.3 has the highest probability of being a carrier: 
 
a) A 
b) B 
c) C 
d) D 
 
8.  ​What  conditional  probability  would  you  consider  in  the  genogram  below  to  calculate  the  risk  of 
woman II.3 being a carrier (according to Bayes’ theorem)? 

 
a) Three healthy sons 
b) A sick brother 
c) The healthy father of II.3 
d) No conditional probability should be considered in this situation 
 
9. Choose the ​INCORRECT​ statement about an autosomal dominant disease that is genetically lethal: 
a) It’s impossible to pass the faulty copy of the gene to the next generation 
b) The alleles with the mutation arise only as a result of de novo mutations 
c) Mosaicism of the parent reduces the risk of the disease manifesting itself in a given family 
d) The risk of the disease recurring is minimal 
 
10. Selection coefficient is: 
a) The percentage of the population with the lethal mutation 
b) The degree of retardation of reproductive capabilities 
c) The amount of infertile individuals due to the mutation 
d) Considered when estimating the risk of lethal genetic conditions 
 
11. Choose the karyotype that shows an unbalanced chromosomal aberration: 
a) 46,XY,inv(5)(q20;q22) 
b) 45,XX,der(13;14)(q10;q10) 
c) 46,XX,del(13)(q12;q13) 
d) All answers are correct  
 
12. 80% penetrance of disease means that in a group of 100 people affected by the disease: 
A. 20% will show symptoms of the disease  
B. 20 people won’t show symptoms of the disease 
C.   80  people  will  manifest  a  full  phenotype  of  the  disease,  while  in  20  people the symptoms will 
be very mild 
D. 20  people  will  manifest  a  full  phenotype  of  the  disease,  while  in  80  people  the  symptoms  will 
be very mild 
  
13. Prader–Willi syndrome is ​most commonly​ caused by: 
A. Deletion of about 4 Mb on the long arm of paternal chromosome 15 
B. Deletion of about 4 Mb on the long arm of maternal chromosome 15 
C. Non-genetic conditions 
D. Abnormal imprinting patterns 
  
14.  Parents  brought  in  their  8  year  old  daughter  into  a  Genetics  Clinic,  as  they  noticed  14  spots  on 
her  skin  in  the  colour  of  “​milk  coffee​”.  The  diameter  of  the  12  spots exceeds 5mm. The patient was 
also diagnosed with optic nerve glioma. 
The last 3 generations of the family have not had any similar symptoms. 
Which clinical diagnosis is the most probable? 
a) Waardenburg syndrome 
b) Warburg syndrome 
c) Waardenburg syndrome type 7 
d) Neurofibromatosis 
  
15. Biologic material used for preparing a cytogenetic specimen for karyotyping ​can not​ be: 
a) amniocytes from amniotic fluid 
b) skin fibroblasts 
c) tumor cells from a specimen fixed in paraffin 
d) fetal tissue 
  
16.  Defect  of  which  gene  or  metabolic  pathway  causes  abnormal  sex  development  in  people  with 
karyotype 46,XX and female androgyny: 
a) SRY gene 
b) Cortisol synthesis pathway 
c) AR gene 
d) SOX9 gene 
  
17.  A  man has been diagnosed with a disease of autosomal dominant pattern. His partner is healthy. 
What kind of genetic advice can be given to the couple?  
a) Probability of having an affected child is ½ 
b) The odds that the couple will have an affected child is 1:2 
c) Each respective child will have a smaller probability of being affected 
d) All sons of this couple will be healthy 
  
18. The chromatogram presents a: 
a) Homozygotic deletion 
b) Heterozygotic deletion 
c) Homozygotic substitution 
d) Heterozygotic substitution 
  
 
  
19.Choose the correct statement regarding linkage disequilibrium: 
A. Linkage  disequilibrium  is  a  non-random  association  of  alleles  in  linked  loci  in  the  whole 
population 
B. In  the  population,  linkage  disequilibrium  between  alleles  increases  with  time  due  to 
recombination 
C. Analysis  of  linkage  disequilibrium  is  often  used  to  specify  the  order  of  the  genes  on  the 
chromosome 
D. All answers are incorrect 
  
20. Choose the genes linked to a high risk of breast cancer: 
a) NF1, APC, BRCA2 
b) BRCA1, BRCA2, TP53 
c) RET, BRCA1, MLH1 
d) PALB2, CHEK2, MSH2 
  
21. A metabolic disorder in an infant is suggested by: 
a) Increased appetite 
b) Often concomitance with a CNS congenital developmental disorder 
c) Sudden deterioration of general state of well-being 
d) All of the above 
  
22.  The  disease  is  inherited  in  an  autosomal  dominant  pattern.  For  the  affected  parent,  2  phases 
have  been  applied.  In  phase  1,  the  disease  categorises  with  marker  2,  in  phase  2  the  disease 
categorises  with  marker  1.  NR  means  no  recombination, R – presence of recombination. Choose the 
correct option describing the 2​nd​ generation (from left to right): 
  
a) Phase 1 NR, NR, R, NR 
b) Phase 2 R, R, NR, R 
c) Phase 1 R, R, NR, R 
d) Both a) and b) are correct 
  

 
 
23. Which of the following properly connects a genetic disease with an abnormal or absent protein? 
a) Duchenne muscular dystrophy - dystrobrevin 
b) SCID - tryptophan deaminase 
c) CF - transmembrane conductance regulator 
d) ADA deficiency - ornithine transcarbamylase 
 
24. An example of epistasis is: 
a) amorphous O allele in ABO group system 
b) phenotype Bombay in H system 
c) phenotype weak D in RHD system 
d) all of the avone are correct 
 
25. Indicate a ​false​ sentence regarding cell-free DNA (cffDNA) 
a) comes mostly from trophoblasts breakdown 
b) its concentration in serum of a pregnant woman increases as the pregnancy progresses 
c) is subjacent to degradation a few weeks after delivery 
d) has a different methylation profile than the mother’s circulating DNA 
 
27.  Pedigree  shows  segregation  of  a  sex-linked  recessive  and  genetically  lethal  disorder.  What  are 
the  odds  and  probability  that  mother  is  a  carrier.  While  making  calculations  assume  that  mutation 
frequency in men (​µm) is 3 times higher than in women (µf). 

 
a) LR = 3, L = 0.667 
b) LR = 4, L = 0.750 
c) LR = 3, L = 0.750 
d) LR = 2, L = 0.667 
 
28.  In  every  generation,  mutated  alleles  present  in  hemizygous  males,  are  eliminated  from  the 
population's gene pool in case of a disease that is: 
a) recessive, X-linked, genetically lethal 
b) fatal recessive, X-linked 
c) only caused by ​ mutations 
d) only caused by inheriting ​ mutation from a father 
 
29. Which sentence concerning the Duchenne muscle dystrophy is true? 
A. In  some  families  occurs  the  germline  Mosaicism,  which  basically  is  the  occurrence  of  a 
mutation of a part of the cells from the germline of the parent 
B. Germline  mosaicism  in  the  Duchenne  dystrophy  doesn’t  have  any  influence  on  the  degree  of 
genetic risk 
C. Germline  mosaicism  is  characterised  by  an  additional  coexisting  random  X  chromosome 
inactivation 
D. Germline mosaicism coexists always with a somatic mosaicism. 
 
30. Rh disease occurs with the situation when in the gene RHD: 
1. In the Mom D/D, in the child d/d 
2. In the Mom D/d, in the child d/d 
A. 1 is correct 
B. 4 is correct 
C. 1 and 2 are correct 
D. 3 and 4 are correct 
 
31.  For a lethal genetic disease, assuming X-linked recessive inheritance, and that µ is the frequency 
of de novo mutation: 
A. The frequency for female carriers is 4µ 
B. The frequency for healthy men is 3µ 
C. The frequency for sick women is 4µ 
D. The frequency for male carriers is 4µ 
 
32. Show the ​WRONG​ answer concerning the whole exome sequencing on the Illumina platform: 
A. During  sequencing,  we  can  observe  a  migration  of  DNA  strands  according  to  the 
electrophoresis rules 
B. Important data files are the files FASTQ and BAM 
C. A  cluster  is  an  aggregation  (agglomeration)  of  DNA  strands,  arising  through  the  bridge 
amplification from one DNA molecule. 
D. The Program IGV serves the visualization of the results 
 
 
33. The phenotype of the Prader – Willi syndrome has the following characteristics: 
A) Low height 
B) Hypotonia 
C) Small feet and hands 
D) All of the above 
 
34.  You  are  consulting  4  pregnant  women.  Which  one  has  the  highest  risk  to  give  birth  to  a  child 
with Down syndrome? 
A) 23-years old woman, who gave birth to a Down syndrome baby two years ago. 
B) 27-years old woman, whose niece has Down syndrome with the karyotype 47, XX, +21 
C) 41-years old woman, whose first child has Down syndrome 
D) 33-years old woman, who is a carrier for the Robertsonian translocation 14;21 
 
35.  Describe  the  risk  for  a  Rh  disease,  when  the  mother  has  the  phenotype  Rh  negative  and  the 
child's father has the genotype D/d in the RHD gene. 
A) 25% 
B) 50% 
C) 75% 
D) 100% 
 
36.  The  grandma  of a proband died of Huntington's disease. This disease is showing 50% penetrance 
in patients at the age of 50 years. 
Which is the probability for that the proband is a carrier of the underlying disease. 
A) ⅓ 
B) ⅔ 
C) ⅙ 

D) ⅛  
 
37. The strategy ex vivo in the genetic therapy is the following: 
A) Allogeneic  transplantation of cells with the aim to rebuild a population of normal cells in the sick 
organism. 
B) Direct application of the transgene to the patients system 
C) Isolation  of  an  adequate  population  of  cells  from  the  patient,  their  modification  and 
reinjection to the patient 
D) Isolation of stem cells from one of the patients tissues and application to a destination, with the 
aim to rebuild the stem cells in the given organ. 
 
38. The definition of penetrance of a given variant (mutation) is the following: 
A) It's  the  probability  to  get  sick  from  the  possession  of the mutation responsible for the given 
disease. 
B) It's the population risk to the development of the disease 
C) Only affects those people, where the genetic disease runs in the family. 
D) Refers to the frequency estimation of a given mutation in a population. 
 
39.  A couple, whose first child died from multiple developmental defects, is getting karyotyping. The 
mother  has  a  normal  karyotype.  The  father  has  a  balanced  reciprocal  translocation  between  the 
chromosomes 4 and 11. You have to inform the parents, that: 
A) Every of their children will have multiple developmental defects 
B) The risk, that this couple will have another child with these defects is high 
C) Balanced  reciprocal  translocations  are  usually  harmless,  so  it  is  unlikely,  that  the  found 
translocation has caused the defects in the child 
D) The chromosomal translocation in the father occured due to dissociation (failure of separation) 
 
40. What's FALSE? aCGH: 
A) Allows the analysis of aberrations throughout the genome in a single hybridization  
B) Allows the identification of balanced aberrations 
C) Is  not  legitimate  in  the  search  for  chromosomal  rearrangements  in  the  area  of the centromere 
and the telomere ​(karyotype analysis) 
D) Allows the diagnosis of biologic material, from which it was impossible to retrieve dividing cells 
 
41. Deformations are related to: 
A) Amniotic band syndrome 
B) Insufficient amount of amniotic fluid 
C) Minor defects of the extremities 
D) the Robin sequence 
 
42.​ Not true ​is that: 
A) with the help of Bayes theorem we're getting the final risk ​+ 
B) LR is for calculating the likelihood ratio ​+ 
C) LR can be bigger than 1 
D) LR allows to directly calculate the risk of a disease 
 
43. Typical clinical problems found in heterozygous Factor 5 Leiden mutation are: 
A) Arterial thrombosis and ischemic stroke in the CNS 
B) Venous thromboembolic disease and early pregnancy termination  
C) Heart attack 
D) All of the above  
 
44. Choose the correct sentence concerning Down syndrome: 
A) Some  children  with  this  syndrome  only  show  small  stature  anomalies  ​108  Milder  clinical 
expression with Mosaicism  
B) The karyotype always shows 47 chromosomes ​(And Robertsonian translocation, 46) 
C) The chromosomal aberration appears just as often in oogenesis as in spermatogenesis  
D) The  karyotype  mostly  shows  triploidy  ​(trisomy,  Downs  is  most  common  autosomal  ​aneuploid 
condition compatible with survival to term) 
 
45. Choose the ​WRONG​ sentence concerning the Sanger sequence for the coding sequence:  
A) It is based on the chromatogram analysis from the thread forward and reverse 
B) The  chromatogram  of  the  patient’s  DNA  is  compared  with  the  chromatogram  of  the  sequence 
analysis (correct translation????) 
C) Basic  information  should  include  the  name  of  the  gene,  the  description  of  the  mutation on the 
level  of  DNA  and  proteins  as  well  as  the  description  if  the  mutation  is  homozygous  or 
heterozygous 
D) The  description  of  the  mutation  always  takes  place  in  the  direction  from  the  sequence 
analysis to the DNA of the patient 
 
46.  Miss  W.  comes  to  you.  She  is  in  the  10​th  week  of  pregnancy  and  has  a  sister  suffering  from 
mucoviscidosis  (cystic  fibrosis)  and  is  worried  that  her  child  will  have  the  disease.  Which  is  the 
probability of being a carrier of the mutated gene CFTR in Miss W.? 
A) ½ 
B) ¼ 
C) ⅔ 
D) 1/25 
 
47. Choose the real definition of genotype: 
A) The genotype is a mutation that occurs in the examined gene in a sick patient 
B) The genotype is the description of the number and structure of all chromosomes 
C) The genotype is a unique allele arrangement in a given locus 
D) The genotype is a unique allele arrangement only on one chromosome 
 
48. Mark the ​WRONG ​sentence about crossing-over​: 
A) Crossing-over  is  the  exchange  of  parts  of  the  DNA  between  homologous  chromosomes 
throughout prophase of the I meiotic division ​+ 
B) Throughout the double crossing-over no recombination appears 
C) The distance between loci is counted in centimorgan ​+ 
D) Loci found on the same chromosome undergo recombination in 50% of cases  
 
49. Show the genotype of the parents, that have the highest risk of Rh disease: 
A) Mom D/d, dad D/D 
B) Mom d/d, dad D/d 
C) Mom d/d, dad D/D 
D) Mom D/D, dad d/d 
 
50. Which of the answers below is correct in the case of spinal bulbar muscular atrophy? 
A) The disease is caused by an excess number of CAG repeats 
B) The correct range of CAG repeats is 9 to 36 ​(11 - 34, info from polish skript) 
C) The correct range of CAG repeats in the polish population is 15 to 94 
D) A and B are correct 
 

2016 FINAL  
1. Indicate true sentence. 
A) Paracentric inversion does not change shape of chromosome. 
B) Robertsonian translocation occurs between two random autosomal chromosomes. 
C) Isochromosomes arise after deletion of two subtelomeric regions. 
D) Direct duplication does not cause change pattern of bands in chromosome. 
 
2. Which sentence describing aCGh microarrays method is true? 
A) Allows to detect majority of known mutations. 
B) Does not detect balanced translocations. 
C) Is used to estimate sex of fetus in very severe diseases linked with chromosome X. 
D) Allows to detect majority of known and unknown mutations. 
 
3. Pedigree shows segregation of autosomal dominant disease with paternal imprinting  (paternal 
allele is inactivated). Which statement is true? 

 
A) The son will be for sure healthy, but daughter will be for sure affected. 
B) The children have the same disease probability which equals 50%. 
C) The children will be for sure healthy but daughter's children can be affected. 
D) The children will be for sure healthy but son's children can be affected. 
 
4.  A  male  with  impaired  hearing  caused  by  a  35delG  homozygous  mutation  in  GJB2  gene  married  a 
woman  who  is  a  heterozygous  carrier  of  the  same  mutation.  What  is the risk of impaired hearing in 
their offspring? 
A) 25% 
B) 50% 
C) 75% 
D) 100% 
 
5. Indicate a karyotype for an unbalanced chromosomal aberration. 
A) 46,XX,del(13)(q12;q13) 
B) 46,XY,inv(5)(q20;q22) 
C) 45,XX,der(13;14)(q10;q10) 
D) A and C 
 
6.  Which  of  the  following  inheritance  patterns  is  relatively  often  associated  with  consanguinity 
between parents? 
A) Autosomal dominant 
B) Autosomal recessive 
C) X-linked recessive 
D) Mitochondrial 
 
7.  The  risk  of  having  a  child  with  Down  syndrome  when  one  parent  is  a  carrier  of  t(21;21) 
translocation is: 
A) 1% 
B) 4%, if translocation is paternal 
C) 25% 
D) 100% 
 
8. On a pedigree, which symbol you will use to indicate a proband? 

 
A) Symbol A 
B) Symbol B 
C) Symbol C 
D) Symbol D 
 
9.  An  individual  has  a  mutation  causing  a  disease  (e.g. retinoblastoma), yet they do not develop any 
symptoms in their lifetime. Which mechanism is responsible for such situation?  
A) Reduced penetrance 
B) Varied expression 
C) Germinal mutation 
D) Locus heterogeneity 
 
10.  When is the most possible inheritance pattern of disease observed in family showed on pedigree 
below? 

 
A) Recessive X-linked 
B) Autosomal dominant 
C) Autosomal recessive 
D) Mitochondrial 
 
11. Which sentence describing Duchenne Muscular Dystrophy is ​false​? 
A) Risk of having second child with DMD by a healthy woman equals 1/3. 
B) Female mutation rate and male mutation rate in DMD are very similar. 
C) Approximately 67% women who have one affected child are carriers of mutation. 
D) Dystrophia gene is mapped on chromosome X. 
 
12. Select the most likely mode of inheritance shown on the pedigree below: 

 
A) Autosomal dominant 
B) Autosomal recessive 
C) Recessive X-linked 
D) Dominant X-linked 
 
13. A clinical genetics laboratory is offering, among its clinical services, a Genetics test that is 
advertised  as  a  test  for  43  genetic  variants  that  are  associated  with  diet  (e.g.  associations  with  fat 
mobilization,  response  to  dietary  nutrients,  increased  waist  circumference,  obesity  risk,  etc.)  and 
exercise  (e.g.  endurance,  power,  speed).  The test is advertised as being useful to guide personalized 
diet and exercise recommendations from nutritional experts for better weight  management 
and  fitness.  An  obese,  Caucasian,  39  years  old  patient  of  yours,  that  you  have  been  guiding on her, 
mostly  unsuccessful,  efforts  to  lose  weight,  asks  your  opinion  about  doing  such  test.  The  patient's 
efforts  to  lose  weight  have  been  unsuccessful  mostly  because  she  does not comply very much with 
the diet recommendations. 

 
A) To  ask  the  laboratory  offering  the  test  about  the studies on which they based their selection of 
the  variants  that  they  test  for,  to  assess  the  applicability  of  the  results  of  the  studies  to  your 
patient. 
B) To  tell  the  patient  that  personalized  medicine  based  on  the genetic makeup of the patient is 
the current trend and thus, since the laboratory is already offering the test and advertising in 
that way, the test will be useful to help her lose weight. 
C) To  ask the laboratory offering the test, if there is any supporting evidence to their claim that the 
test  results,  and  the  consequent  personalized recommendations, will improve the chances that 
patients  will  lose  weight,  in  comparison  with  current,  non-personalized,  interventions  for 
weight loss. 
D) To  expect  that, most likely, the evidence described in C is still not available, because appropriate 
randomized  controlled  trials,  considered  the best providers of such evidence, are expensive and 
take time to accomplish. 
 
14.  Continuation  of  the  above.  The  patient  decided  to  go  ahead  and  take  the  test. It was found that 
she  carries  the  less  common  variant  allele  at  a  gene  named  FTO.  Carriers  of  such  less  common 
variant  allele  have been found, in a single study, published in 2012, to have better response (1.51 kg 
greater  reduction  in  weight)  to  a  „low  calorie,  high  protein  diet“  than  to  a  „low  calorie,  low  protein 
diet“  after  controlling  for  age,  sex,  ethnicity  and  baseline  BMI.  The  nutritionist  provided  by  the 
laboratory,  based  on  this study, proposed a diet to the patient that was low in calories and relatively 
high in protein. 
You  did  some  further  research  and  found  a  few  other  studies  that  do  not  seem  to  corroborate  the 
effect of the above referred study, although they had different designs.   
Based on the above, which of the following sentences is correct? 
A) It is almost sure that the patient will lose weight with this new diet. 
B) It is almost sure that the patient will not lose weight with this new diet. 
C) It is difficult to predict what will happen with the weight of this patient with the new diet. 
D) The patient should not try this new diet and should give up on trying to lose weight. 
 
15. Recognizable genetic etiology concerns all possible anomaly patterns ​except​: 
A) Dysplasia 
B) Association and syndrome 
C) Disruption and deformation 
D) Sequence 
 
16.  What  should  be  looked  for  when  collecting  medical  history  for  a  patient  with  possible  genetic 
disease? 
A) Miscarriage 
B) Ethnic origin 
C) In vitro fertilization 
D) All of the above 
 
17. Indicate karyotype with unbalanced chromosomal aberration 
A) 46, XX, del (13) (q12; q13) 
B) 46, XY, inv (5) (q20; q22) 
C) 45, XX, der (13; 14) (q10;q10) 
D) Correct A and C 
 
18.  A  39  years  old  patient  of  yours  is  pregnant.  In  her  reproductive  history,  you  see  that  she  had  a 
miscarriage  around  10  weeks,  in  her  first  pregnancy,  then  she  had  a  healthy  daughter,  currently  7 
years  old,  and  then  a  second  miscarriage  around  a  similar  gestational  age.  You  referred  her  for 
genetic  counseling.  After  counseling,  she  decided  to  have  the amniocentesis to check the karyotype 
of the fetus. 
 
A) Because  she  understood  that,  because  of  her  age  she  has  a  risk  for  fetal  chromosomal 
aneuploidies that is above the average and high enough to cause concern. 
B) Because  she  understood  that,  because  of  her  age  she  has  a risk she had an increase in the risk 
for fetal structural chromosome anomalies. 
C) Because  she  understood  that,  because  she  had  2  miscarriages  she  has  an  increase  in  the  risk 
for fetal structural chromosome aneuploidies. 
D) Because  she  understood  that she has a risk for chromosomal aneuploidies, which is common in 
every woman, regardless of age. 
 
19.  Continuation  of  the  above.  After  approximately  10  days,  she  calls  you  very worried because the 
laboratory  called  her  to  tell  her  that  the  fetus,  has  a  balanced  robertsonian  translocation  between 
the  chromosome 21 and the 13 and one chromosome X and one chromosome Y, which correspond to 
a  karyotype  45,  XY,  ​…  (22;23)  (q10;q10).  She  was  referred for genetic counseling with a genetical ​… 
the  earliest  appointment  was  possible  only  within  3  days  and  she  is  very  worried,  despite  the  fact 
that the laboratory staff person that called her, told her not to be worried.   
At this stage, with just this information available, which of the following statements is not correct? 
A) The  fetus  will  not  have  an  abnormal  phenotype  caused  by  the  genomic  defect,  because  the 
translocation is balanced and the breakpoints affect non-essential loci. 
B) The  patient  and  her  partner,  assumed  to  be the father of the baby, will need to check their own 
chromosomes  by  karyotype  as  verify if the fetus inherited the anomaly from one of the parents 
or if it was a de novo defect. 
C) If  one  of  the  parents  is  also  a  carrier,  her  daughter  may  then  be  a  carrier  and  thus,  should  get 
genetic  counseling  when  she  reaches  reproductive  age.  This is important because, if she is also 
a carrier of the same defect, she  will also have reproductive risks. 
D) The  only  problem  is  for  the  future  reproductive  plans  of  the  individual  that  will  result  from 
this  pregnancy.  There  will  be  a  risk  for  live  newborns  with  trisomy  21,  trisomy  13  or 
monosomy 13 or monosomy 21. 
 
20. Fabry's disease manifests with: 
A) Burning pain in the palms and soles precipitated by stress, alcohol, exercise or heat, 
B) Neonatal coma and convulsions, 
C) Rapid developmental regression, 
D) Mental retardation. 
 
21.  Abdominal  ultrasound  done  in  an  otherwise  healthy  neonate  reveals  a  unilateral  multicystic 
kidney. What is the most likely karyotypic finding in this individual? 
A) 47, XXY 
B) 45, X 
C) 46, XY 
D) 46, XY ish del (7)(q11.23q11.23)(ELN-) Williams 
 
22.  A  useful  diagnostic  marker  leading  to  a  possible  diagnosis  of  a genetic syndrome in a child with 
intellectual disability (ID) is: 
A) Macrocephaly / Microcephaly 
B) Facial dysmorphism 
C) Accompanying behavioral abnormalities 
D) All of the above 
 
23. An „all or none“ phenomenon referring to the presence or absence of observable phenotypic 
expression  of  features  of  a  dominant  disease  in  an  individual  known  to  have  a  mutant  allele  is 
called: 
A) Variable expression 
B) Incomplete penetrance 
C) Phenocopy 
D) Dysplasia 
 
24.  One  of  your  patients,  a  38  female,  is  pregnant  for  the  second  time.  She had a girl before, now 7 
years  old.  She  consults  you  because  of  common  cold.  You  ask  her  how  is  the  pregnancy  going.  She 
says  she  is  happy  because  she  is  going  to  have  a  boy.  She  knows  that  because  she  did  a  fetal 
karyotype  reported  as  „normal  male,  46,  XY“.  She  expresses  concern  only  with  the  fact  that  during 
the  ultrasound  exams  the  doctor  was  never  able  to  see  the  sex  of  the  baby,  because  of  positioning 
reasons.  At  birth  the  patient  is  congratulated  on  the  birth  of  an  apparently  healthy  girl!  She  gets 
puzzled. Which of the following sentences is appropriate? 
A) It is impossible to have a XY karyotype and be a girl. 
B) If  the  sex  chromosomes  are  confirmed  to  be  XY,  one  of  the  possible  explanations  for  the 
discrepancy  between  the  sex  and the karyotype is a rare condition in which the SRY gene which 
codes  for  a  protein  named  Testis  Determining  Factor,  a  gene  normally  located  in  the  Y 
chromosome, may have been translocated in an autosome. 
C) If  the  sex  chromosomes  are  confirmed  to  be  XY,  one  on  the  possible  explanations  for  the 
discrepancy  between  the  sex  and  the  karyotype  is  a  rare  condition,  which  is  X-linked 
recessive,  and  is  caused  by  an  inactivating  mutation in the gene that codes for the androgen 
receptor,  leading  to  a  complete  absence  of  the  receptor,  a  condition  known  as  Complete 
Androgen Insensitivity Syndrome (CAIS).  
D) There  is no risk that the sister is affected with the same condition of the newborn, regardless of 
the cause of the sex reversal observed in the newborn.  
 
25.  A  male  patient  of  yours  brings  you,  his  primary  care family physician, his 4 years old male child, 
who  has  symptoms  of  a  common  cold.  You  know  the  child  has  been  seen  in  specialized  pediatric 
services  because  he  was  late  to  walk,  and  he  does  not  speak  yet,  having  been  diagnosed  with 
intellectual  disability.  You  also  have  noted  that  the  child  has  some  peculiar  face,  which  you,  a 
non-specialist,  have  some  difficulties  in  describing.  You  had  been  told  before  that  the  child  had  a 
karyotype  done  1  year  ago  and  it  had  been  reported  as  normal.  The  father  tells  you  that  the 
pediatrician  had  sent  the  child  for  a  clinical  geneticist  consultation  and  that  the  geneticist,  after  a 
detailed observation and description, had suggested to perform “chromosome microarray”.  
Which of the following opinions about this suggestion by the clinical geneticist is correct? 
A) If  the  karyotype  was  normal,  a  chromosomal  microarray  will  not  bring  any  more  relevant 
information and so it is a waste of time and money.  
B) The  geneticist  is  probably  expecting  to  identify a balanced translocation, a chromosomal defect 
not  suitable  for  detection  by  karyotype,  which  is  a  likely  explanation  of  the  phenotype  of  the 
child.  
C) The  geneticist  is  probably  expecting  to  find  a  submicroscopic  genomic  unbalance  that could 
be the explanation of the phenotype of the child.  
D) Chromosome  microarrays  are  still  not  recommended  as  clinical  exams.  They  are  still only used 
for research.  
 
26. The epigenetic changes differ from other DNA changes found in cancers because they are: 
A) Silent 
B) Rare 
C) Nonsense 
D) Reversible 
 
27.  According  to  the  Knudson  hypothesis,  known  also as two hits hypothesis, phenotypic effect of a 
cancer suppressor gene damage appears after:  
A) Inactivation of both alleles 
B) Demethylation of both alleles 
C) Amplification of both alleles 
D) The correct A and B  
 
28.  What  is  the  estimated  number  of  women  carriers  of  a  X-linked  recessive  disease  in  40  mln 
population if prevalence of affected men equals 1/10000? Ignore de novo mutation phenomena.  
A) 4 thousand 
B) 8 thousand 
C) 16 thousand 
D) 64 thousand 
 
29.  A  23 years old male who you have diagnosed with diabetes a few years ago, tells you that he has 
been  having  hearing  problems  which  you  confirm using appropriate hearing tests. Later he tells you 
that  he  had  two  episodes  of  seizures  and  an  electroencephalogram  confirms  the  diagnosis  of 
epilepsy.  Due  to  the  combination  of  progressively  acquired  symptoms  in  different  organs  and 
systems,  that  are  especially  demanding  in  terms  of  cell  energy  requirements  you  consider  the 
hypothesis  of  a  mitochondrial  disorder.  You  refer  your  patient  to  a  specialist  who,  after  requesting 
several  types  of  tests,  later  confirms  the  diagnosis.  Genetic  testing  identifies  a  mutation  in  the 
mitochondrial DNA. The patient consults you for further information.  
Of  the  following  statements  about  these  type  of disorders (grouped under the name “mitochondrial 
disorders”), which one is ​incorrect​?  
A) All mitochondrial disorders are caused by mutations in mitochondrial DNA.  
B) Several  of  these  disorders  tend  to  be  progressive  and  it  may  be  possible  that  symptoms  in 
other systems may still appear.  
C) There  is  no  specific  treatment  of  these  conditions  and  each  problem/symptom  must  be 
managed as they would be in people who have them because of different etiologies.  
D) Among  individuals  with  the  same  mutation  in  the  mitochondrial  DNA  the  symptoms  and 
evolution of the disease have a relatively broad range of severity.  
 
30.  Continuation  of  the  above.  The  patient  described  in  the  previous  question  is  worried  about  the 
possibility  of  having  affected  children.  You  ask  him  about  his family history and he says that he has 
a  younger  sister,  aged  17.  His  parents  (father  aged  50  also  complaining  of  hearing  problems 
recently,  and  mother  aged  45,  has  been  relatively  healthy,  although  lately  she  refers  feeling  tired 
easily).  
About his family, which of the following sentences is correct?  
A) There is a small risk of transmitting the condition to his daughters but not to his sons.  
B) There is a small risk of transmitting the condition to his sons but not to his daughters.  
C) His  father’s  deafness  may  also  have  the  same  cause  of  his  own  symptoms,  that  is,  the  father 
may  also  have  the  same mutation is his mitochondrial DNA, and the patient may have inherited 
it from him.  
D) His  mother’s  symptoms  of  easily  feeling  tired  may  also  have  the  same  cause  of  his  own 
symptoms,  that  is,  the  mother  may  also  have  the  same  mutation  is  his  mitochondrial  DNA, 
and the patient may have inherited it from her.  
 
31.  Assuming  a  dominant  X-linked  inheritance  pattern,  penetrance  of  100%  and  a  frequency  of 
mutated  allele  of  0.2,  the  likelihood  of  the  pedigree  (the  disease  occurrence  in  the family of four as 
on the scheme) is approximately: 

 
A) 0.032 
B) 0.064 
C) 0.128 
D) 0.2 
 
32.  What  are  the  odds  (likelihood  ratio,  LR)  that  a  tested  marker  is  linked  (theta  =  0)  to  a  gene 
causing  an  autosomal  dominant  disease  in  the  family  showed  on  the  right.  Results  of  the  genetic 
marker genotyping are indicated on the pedigree.   

   
A) 1 
B) 16 
C) 32 
D) 64   
 
33. Which of the following statements regarding the autosomal dominant diseases is ​not true​? 
A) These diseases manifest themselves in those heterozygous for the mutated gene.  
B) The  probability  of  the  disease  in  a  relative  affected  person  may  be  dependent  on  the 
penetration.  
C) The disease affects many people, but only in one generation.  
D) None of the parents of a affected person has to be heterozygous mutated gene 
 

34. B 
35. C 
 
 

 
 

2016 FINAL Retake  


 
1-5 not visible 
 
6.  Which  of  the  following  symptoms  is  not  present  in  individuals  with  Klinefelter  syndrome 
(47,XXY)? 
a) Tallness 
b) IQ lower than in healthy siblings 
c) Webbed neck 
d) Infertility 
 
7. Mark a false statement. 
a) Females with FRA X usually have severe disability 
b) Females with FRA X may experience premature menopause 
c) Prenatal diagnosis in FRA X syndrome is possible 
d) In males with FRA X, delayed speech development occurs 
 
8. The risk of recurrence of multifactorial disease does not depend on: 
a) Frequency of the particular disease in the population 
b) Sex 
c) Number of miscarriages 
d) Number of affected individuals in the family 
 
9.  A  child  with  an  autosomal  dominant  disease  was  born  to  healthy  parents.  What  is  the  risk  that 
their next child also will be affected? 
a) 100% 
b) 0% 
c) 50% 
d) The risk will depend on whether one of the parents is a germinal mosaic 
 
10. The symbol shown here denotes: 

a) Siblings  
b) Monozygotic twins 
c) Dizygotic twins 
d) Consanguineous relationship 
 
11. Indicate a karyotype for an unbalanced chromosomal aberration 
a) 46,XX,del (13)(q12;q13) 
b) 46,XY, inv(5)(q20;q22) 
c) 45,XX,der(13;14)(q10;q10) 
d) A and C 
 
12. Select the most likely mode of inheritance shown on the pedigree below: 
a) Autosomal dominant 
b) Autosomal recessive 
c) Recessive X-linked 
d) Dominant X-linked 
 
 
 
 
 
13. Recognizable genetic etiology concerns all possible anomaly patterns except: 
a) Dysplasia 
b) Association and syndrome 
c) Disruption and deformation 
d) Sequence 
 
14.  An  “all  or  none”  phenomenon  referring  to  the  presence  or  absence  of  observable  phenotypic 
expression  of  features  of  a  dominant  disease  in  an  individual  known  to  have  a  mutant  allele  is 
called: 
a) Variable expression 
b) Incomplete penetrance 
c) Phenocopy 
d) Dysplasia 
 
15. Indications for genetic evaluation, including testing, should include individuals with: 
a) Autism and developmental regression 
b) Autism  and  any of the following: dysmorphism, congenital anomaly (-ies), developmental delay, 
macrocephaly > 3SD, microcephaly <3SD 
c) Autism  and  family  history  of  autism  or  other  neuropsychiatric  disease  in  1st/2nd  degree 
relatives 
d) All of the above 
 
16.  After  an  uneventful  pregnancy,  a  31  years  old  woman  delivers  at  term  a  newborn  by  normal 
vaginal  delivery.  The  mother  asks  anxiously  about  the  “sex  of  the  baby”.  It  seems  that  there  had 
been  some  issues  with  the  ultrasound  assessment  of  the  sex,  which  was  always  difficult  and 
apparently  seeming  to  contradict  the  results  of  a  cell-free  maternal  plasma  screening  test  for 
chromosomal  defects  that  the  patient  had  decided  to  have.  The  test  reported  a  46,XX  fetus but the 
doctor  doing  the  ultrasound  scans  thought  it  could  be  a  boy,  although  he  could  not  be  sure.  The 
nurse  attending  to  the  baby  looked  at  the  genitals  of the baby and started mumbling with the other 
colleagues  instead  of  answering  the  mother’s  question.  They  took  the  baby  away  and  called  the 
attending  doctor.  A few minutes later the doctor came in and told the patient that the genitals of the 
baby  were  ambiguous  and  it  was  not  possible  to  determine  the  sex  at  that  moment.  Some  tests 
would  need  to  be  ran  first.  A  few  anguishing  days  later  the  mother  was  told  the  baby  had  a  46XX 
karyotype  but  the  genitals  had  an  abnormal  development  due  to  an  autosomal  recessive  heritable 
monogenic  condition  called  congenital  adrenal  hyperplasia,  that,  in  this particular case, was caused 
by  a  complete  deficiency  of  one  enzyme  named  21-hydroxylase.  Of  the  following  sentences,  which 
ones are correct? 
1) In this condition,the gonads are normal ovaries 
2) The  internal  sexual  reproductive  organs  have  normal  female  organs  (uterus  and  tubes)  but 
also some Wolffian duct structures 
3) Other  than  the  external  genital  issue  and  the  sexual  development  anomaly,  there  are  no other 
medical care requiring issues. 
4) This condition has a likely 50%risk of recurring in each of the pregnancies of the couple 
5) The baby, when grown up, and willing to have children, may require medical assistance 
6) If  reproduction  is  achieved  by  the  grown  up  baby,  there  will  be  a  50%  risk  of  transmitting  the 
disease to the offspring, regardless of the genetic makeup of the partner. 
 
a) 1,2,5 
b) 1,4,6 
c) 2,3,6 
d) 2,3,5 
 
17.  A  woman  of  40  years  and  her  20  years  old daughter got pregnant more or less at the same time 
and  they  both  miscarried.  The  mother  ,  at  12  weeks  and  the daughter at 9 weeks. This was the first 
pregnancy  of  the  daughter  and  the  4th  of  the  mother,  who  had  another  miscarriage  at  around  8 
weeks, on her third pregnancy, when she was 38 years old.  
Which of the following sentences is correct? 
a) The  most  likely  cause  of  the  miscarriages  of  the  mother  is  random  accidental  aneuploidy 
(trisomies) but the miscarriage of the daughter is more likely to have another cause. 
b) The  most  likely  cause  of  the  miscarriage  of  the  daughter  is  random  accidental  aneuploidy 
(trisomies)  but  the  miscarriages  of  the  mother  are  more  likely  to  be  caused  by  another  cause, 
such as unbalanced translocation. 
c) The  most  likely  cause  of  the  miscarriages  of  both,  mother  and  daughter,  is  random 
accidental aneuploidy (trisomies) 
d) The  most  likely  cause  of  the  miscarriages  of  both,  mother  and  daughter,  is  unbalanced 
translocations. 

 
 
18.  On  the  basis  of  a  drawing  showing  chromatograms  of  the  STR  marker  analysis,  point,  in  which 
meiotic division and in which parent nondisjunction occured. 
a) In the mother in the first meiotic division 
b) In the father in the first meiotic division 
c) In the mother in the second meiotic division 
d) In the father in the second meiotic division 
 
CANT SEE 19-20 
 
21. Fabry's disease manifests with: 
A. Burning pain in the palms and soles precipitated by stress, alcohol, exercise or heat, 
B. Neonatal coma and convulsions, 
C. Rapid developmental regression 
D. Mental retardation. 
 
23. Point one of the following karyotypes that is the​ least likely ​to involve at a live newborn: 
A) 45,X 
B) 46, XY,-16qh 
C) 47,XX,-7 
D) 47,XX,-11 
 
24. Spinal muscular atrophy: 
a) Is a heterogenous group of disorders 
b) Approximately 95% of 5MA affected patients lack both copies of SMN1 gene (unclear) 
c) Is caused by mutations in SMN2 gene 
d) Is inherited as an X-linked trait 
 
25. Which woman is the greatest risk of having a baby with down’s syndrome? 
a) 25-year old woman who had a baby with karyotype 47,XX,-21 
b) 30-year old woman who is a carrier of the Robertsonian translocation 21;21 
c) 35-year old woman whose niece has Down syndrome 
d) 40-year old woman who had a baby with karyotype 47,XY,-21 
 
26. Select which statement regarding autosomal recessive inheritance is ​incorrect​. 
a) If both parents are carriers, then the fertilization risk that their child will be a carrier is 2/3 
b) These diseases are more common in societies where marriage between relatives are frequent  
c) The incidence of these diseases in men is the same as in women 
d) The disease usually occurs in one generation 
 
27. The woman, carrier of a X-linked recessive disease, sometimes reveals symptoms of the disease, 
which results from the phenomena: 
a) Variable expression 
b) Mitochondrial heredity 
c) Inactivation of one of the X chromosome​ (Lyonization effect) 
d) Incomplete penetration 
 
28. The most common cause of intellectual disability in the population is: 
a) Trisomy 21 
b) Trisomy 13 
c) Trisomy 18 
d) Turner syndrome 
 
29. Which statement(s) about the frequency of mutations in germ cells are true?  
a) Chromosomal mutations occur more frequently in women 
b) Point mutations occur more frequently in men 
c) In both sexes mutations in the germ cells arise more frequently in the elderly 
d) All of the above 
 

30. Person 3:  
a) Is currently healthy but with high risk will be ill in the future 
b) Is a carrier of the autosomal recessive mutation 
c) Exhibits the subclinical characteristics of the disease 
d) Is a carrier of the X-linked recessive mutation 
 
31. Lod score = 0 means that: 
a) Is 10 times less likely that loci are linked than they are independent  
b) Is 10 times more likely that loci are linked than they are independent 
c) Is just as likely that loci are linked and they are independent 
d) None of the above 
 
32.  The  gene  responsible  for  the  occurrence  of  family  polyposis  colon  cancer  (a  form  of  hereditary 
dominant colorectal cancer) is: 
a) Tumor suppressor gene 
b) Proto-oncogene 
c) DNA repair gene 
d) Gene of the reduced-shortening telomeres in dividing cells 
 
33.  A  25  year  old  woman  two  days  ago  gave  birth  to  a  healthy  baby.  Her  mother  (the  child’s 
grandmother)  as  a  child  was  operated  on  due  to  retinoblastoma.  The  woman  never  became  ill  with 
the  cancer,  but  she  wants  to  know  what  is  the  probability  of  occurrence  of  the  disease  in  the 
newborn  child.  Both  the  father  and  grandfather  of  the  child  are  homozygous  wild  types. In order to 
determine the genetic risk you will take into account: 
a) The incident of disease in the population 
b) That there is no maternal disease 
c) That there is no disease in child 
d) The correct are B and C 
 
34. The most complete interpretation of the karyotype test result 47,XXY [60]/46,XY [40] is: 
a) Mosaicism-  in  60  of  the  100  examined  cells  identified  karyotype  of  Turner  syndrome,  other 
normal cells 
b) Mosaicism- in 60% identified the karyotype of Klinefelter syndrome, other cells -normal 
c) Mosaicism  -  60%  of  the  100  examined  cells  with  karyotype  of  Klinefelter  syndrome,  other 
normal cells 
d) None of the above 
 
35.  Pedigree  shows  the  segregation  of  Duchenne dystrophy. What are the odds that the mother in a 
carrier of pathogenic mutations? 

 
a) 1 
b) 1:2 
c) 2:1 
d) None of the above 
 
 

MCQ-1  
 
1. Which of the following is inherited as an autosomal recessive condition?  
a) Achondroplasia 
b) Phenylketonuria 
c) Factor VIII deficiency (haemophilia A) 
d) Neurofibromatosis  
e) Spina bifida 
 
2.  A  36-  year  old  woman  has  only  a  son  in  whom  haemophilia  has  diagnosed.  He  is  also  affected. 
Which of the following statements is correct?  
a) The condition is X-linked dominant 
b) All of her future daughters will be carriers 
c) All of her sons daughters will be carriers 
d) Her son’s sons will be at 50% risk of haemophilia 
e) Fetal diagnosis of haemophilia for future pregnancy by DNA analysis is most used 
 
3.  A  man  and  a  woman  have  short  stature,  prominent  forehead,  shallow  nasal  bridge,  short  limbs, 
and  other  skeletal  changes.  Both  of  them  have  been  told  that  they  have  achondroplasia  (100800), 
and  wish  to  know  their recurrence risk for affected children. Match the recurrence risks for the child 
of achondroplastic parents to be affected (heterozygous and homozygous). 
a) 100% 
b) 75% 
c) 50% 
d) 25% 
e) Virtually 0 
 
4.  Following  the  birth of an affected child, the probability for the second child of the achondroplastic 
parents to be affected.  
a) 100% 
b) 75% 
c) 50% 
d) 25% 
e) Virtually 0 
 
5. The cytogenetic term “6q+’’ refers to… ? 
a) 46,XX,dup(6q) 
b) Extra chromosome material derived from the long arm of chromosome 6 
c) 46,XX,dup(6) 
d) Extra chromosome material, origin specified, attached to the long arm of chromosome 6 
e) 47,XX,+6 
 
 
6. Mark the true statement regarding the family tree shown below: 

 
 
a) I-2 is dead  
b) II-2 is a male 
c) III-3 is affected 
d) III-1 and III-3 are consanguineous couple 
e) II-3 has one boy 
 
7. Mark the ​FALSE​ statement regarding the family tree shown below: 

 
a) The condition is X- linked 
b) II-5 is a non-penetrant gene carrier 
c) There is a 50% risk that any offspring of II-3 will carry the faulty 
d) The condition is likely to affect males and females equally 
e) II1-1 AND III-2 have 50% of their genes in common  
 
8. Which of the following statements on birth defects is true?  
a) Occur in 3:1000 live births  
b) In most children involve multiple organs 
c) Are common in chromosomal abnormalities 
d) Are almost all due to known single- gene defects 
e) Always involve malformations  
 
13. Which of the following conditions have polygenic inheritance? 
a) Galactosaemia 
b) Polydactyly 
c) Meningomyelocele 
d) Turner’s syndrome 
e) Marfan’s syndrome  
 
14. What proportion of early spontaneous abortions are chromosomal? 
a) 1% 
b) 5% 
c) 10% 
d) 25% 
e) 50% 
 
15. Anticipation is characteristic of conditions caused by  
a) Microdeletions 
b) Mitochondrial inheritance 
c) Genomic imprinting 
d) Trinucleotide repeat expansions 
e) Germline mosaicism  
 
 
 

2015 4yr  
1) A male has 2 sons affected by a dominant X-linked disease: 
a. His all children are likely to be affected 
b. His all sons are likely to be affected 
c. His all daughters are likely to be affected 
d. Other 
 
2) THE BEST description of the term ‘syndrome’ is: 
a. A chromosomal abnormality, e.g Down syndrome  
b. A  set  of  developmental  anomalies  occurring  together  in  an  recognizable  and  consistent 
pattern and assumed to be of single etiology 
c. Two congenital anomalies in one individual  
d. An abnormal developmental process 
 
3) Frequency of an allele on X chromosome is 10%. At least one copy of his allele will be carried by 
a. 10% population 
b. 10% males 
c. 19% females 
d. B and C 
 
4) Maternal uniparental disomy (UPD)  
a. Is always symptomless if mother is healthy and has normal karyotype  
b. May cause pathology if the involved chromosome contains loci with maternal imprinting 
c. May cause pathology if the involved chromosome contains loci with paternal imprinting 
d. B and C 
 
5)  Based on the analysis of STR genetic markers carried out in trisomic healthy child and his parents, 
you  find  that  nondisjunction  occurred  during  (peak  position  on  the  horizontal  axis  corresponds  to 
the type cariant, the peak height corresponds to the number of copies): 
 
 
 
 
 
 
a. Spermatogenesis, in the first meiotic division 
a. Spermatogenesis, in the second meiotic division 
b. Oogenesis, in the first meiotic division 
c. Oogenesis, in the second meiotic division 
 
6) Symbol beneath denotes 
a. Perinatal death  
b. Infertility  
c. Lack of children 
d. Other 
 
7) In achondroplasia (inheritance autosomal dominant) which is true? 
a. The risk to the sibling of an affected person who has normal parents is 50% 
b. The risk of achondroplasia each offspring of an affected person is 50% 
c. There is an increased frequency of consanguinity in the parents of affected peoples 
d. All the offspring of two achondroplastic parents will be affected 
e. The abnormal gene is on the X chromosome 
 
8)  Pedigree  shows  a  family  in  which  there  was  a  case  of  with  a  boy  with  Duchenne  muscular 
dystrophy. What are the odds that mother is a carrier of pathogenic mutation 
a. 1:1 
b. 1:2 
c. 1:3 
d. 2:1 
 
 
9) Prenatal diagnosis in Duchenne muscular dystrophy  
a. Is  available  to  carriers  of  a  known  dystrophin  mutation  by  chorionic  villus  sampling (CVS) at 24 
weeks gestation  
b. Is  available  to  carriers  of  a  known  dystrophin  mutation  by  chorionic villus sampling (CVS) at 
11 weeks gestation 
c. Is available only to patients without gonadal mosaicism 
d. Is not available to mothers of an apparently de novo DMD mutation (germline mosaicism) 
 
10) Please indicate a true risk for multifactorial disorders: 
a. Cleft/lip palate 1%, spina bifida 10%, schizophrenia 50% 
b. Cleft/lip palate 4%, spina bifida 4-5%, schizophrenia 10% 
c. Cleft/lip palate 12%, spina bifida 12%, schizophrenia 20% 
d. Cleft/lip palate 1%, spina bifida 10%, schizophrenia 20% 
 
11) Which percentage of abnormalities is observed also in first degree relatives of a patient 
a. Mental  retardation:  severe  25%,  mild  35%,  recessive  disorders  10%,  congenital  malformation 
10% 
b. Mental  retardation:  severe  35%,  mild  25%,  recessive  disorders  50%,  congenital  malformation 
20% 
c. Mental  retardation:  severe  50%,  mild  35%,  recessive  disorders  10%,  congenital  malformation 
10% 
d. Mental  retardation:  severe  25%,  mild  40%,  recessive  disorders  10%,  congenital  malformation 
10% 
 
12) Please indicate false opinions concerning genetic counseling 
a. Not needed if a single affected child present 
b. Not needed if disease is not inborn (no symptoms from the birth) 
c. Not  needed  in  a  case of autosomal recessive disease in a second child if the first child is already 
affected 
d. All the opinions (A,B,C) are false 
 
13) Which causative associations between clinical syndrome and chromosomal aberration are true? 
a. Wolf-Hirschhorn 4p, Cri-du-chat 5p, Smith Magenis del117p11.2 
b. Wolf-Hirschhorn 5p, Cri-du-chat 4p, Smith Magenis del117p11.2 
c. Wolf-Hirschhorn 4p, Cri-du-chat 5p, Smith Magenis dell116p11.2 
d. Wolf-Hirschhorn 4p, Cri-du-chat 5p, Smith Magenis del117q11.2 
 
14) Emery-Dreifuss muscular dystrophy is characterized by: 
a. Selective distal muscle wasting 
b. Limited neck and trunk flexion, limitation of elbow extension  
c. Sparing of forearm muscles 
d. B and C are true 
 
15) Duchenne muscular dystrophy is caused by: 
a. 60% large deletions, 35% point mutations, 5% duplications 
b. 60% large deletions, 20% small duplications 
c. Point mutations in DMD gene 
d. Mutations in the DMD20 gene 
 
16) Spinal muscular atrophy: 
a. Is a heterogenous group of disorders 
b. Approximately 95% of SMA affected patients lack both copies of SMN1 gene  
c. Is caused by mutations in SMN2 gene 
d. Is inherited as an X-linked trait 
 
17) Gonadal mosaicism in Duchenne muscular dystrophy  
a. Has been never found in muscular dystrophies 
b. Is present in 80% of DMD affected patients 
c. Occurs in some DMD affected patients 
d. Should be considered in all DMD affected patients 
 
18)  A  baby born with squashed face and talipes equinovarus because of large fibroids in its mother’s 
uterus would be classified as having: 
a. A malformation 
b. A deformation 
c. A dysplasia 
d. A sequence 
e. A syndrome 
 
19) Which of the following statements on birth defects is true? 
a. In most children involve multiple organs 
b. Are common in chromosomal aberrations 
c. Occur in 3:1000 live births 
d. Are all due to known monogenic defects 
e. Always involve malformations 
 
 

POLISH DIVISION 2017 


1. Proband: 
a. Carrier 
b. Parent 
c. Propositus 
d. Interesting case 
 
2. Which  of  the  following  situations  present  the  highest  risk  of  the  child  to  have  Down 
syndrome? 
a. 45 year old woman 
b. 25 year old woman, pregnant before, child with Down syndrome 
c. 20 year old woman, carrier of balanced translocation of chromosomes 14 and 21 
d. 20 year old male, carrier of balanced translocation of chromosomes 23 and 21 
 
3. In  the  first  pregnancy,  the  boy  with  DiGeorge  syndrome  was  born.  What  is  the  chance  that 
that syndrome will appear in the next pregnancies? 
a. 50% for male gender 
b. 50% independently from the gender 
c. It is population risk 
d. All of the answers are false 
 
4. Pleiotropic gene: 
a. Is expressed only in one particular tissue 
b. Is present in the genome in many copies 
c. Influences more than one trait 
d. Due to its size it is more prone for mutations 
 
5. Which of the following chromosomes never takes part in Robertsonian translocation: 
a. 13 
b. 15 
c. 16 
d. 21 
 
6. Trait,  which  would  explain  the  occurrence  of  inappropriate  phenotype  in  people  with 
inappropriate sex chromosome number : 
a. Inactivation of only maternal X chromosomes 
b. Inactivation of only paternal X chromosomes 
c. Incomplete  inactivation  of X chromosomes at the ends of short and long arm (active fragments 
on both copies) 
d. None of above 
 
7. In  the  familial  hypercholesterolemia,  the  mutation  in  LDL  receptor  gene  was  found.  Both 
homozygotes  and  heterozygotes  are  affected.  However,  the  symptoms  in  homozygotes  are 
more severe. Molecular basis of this disease is: 
a. Negative dominant effect 
b. Haplotype insufficiency 
c. Gain of function 
d. Positive dominant effect 
 
8. On  the  basis of segregation analysis, the following scheme presents disease that is inherited 
by: 
a. Autosomal dominant 
b. Autosomal recessive 
c. Dominant X linked 
d. Recessive X linked 
 
1. Down syndrome (the most common mutation): 
a. Aneuploidy 
b. Polyploidy 
c. Triploidy 
d. Somatic mutation 
 
9. The  following  profile  of  gene  mutation  in  neoplasms  indicates  that  gene  mutated  this  way 
may act in the neoplastic process as: 
a. Passenger gene 
b. Suppressor gene 
c. Mutagen 
d. Oncogene 
 
10. Angelman syndrome is a result of: 
a. One parent disomy of maternal chromosome 15  
b. Deletion of maternal fragment of chromosome 15 (del15q11-13) 
c. Deletion of paternal fragment of chromosome 15 (del15q11-13) 
d. None of the above 
 
1. What  is  the  genetic  risk  for  child  whose  one  parent  is  affected  with  autosomal  dominant 
disease (heterozygote) and other is healthy: 
a. For each child it is equal and it is ½ according to the rule of independence 
b. ½ but only for daughters 
c. ½ for the first child of this pair, with each next one the risk is lower 
d. Impossible to assess the risk based on this information 
 
1. The advantage of analysis of chromosomes in metaphase after staining for bands G ​is not​: 
a. Possibility of assessing whole karyotype in single test 
b. Possibility of using both chromosomal number aberrations and structural aberrations 
c. Possibility of using microdeletions 
d. Possibility of using Robertsonian translocations 
 
1. Which statement concerning Duchenne dystrophy is correct: 
a. Female carriers of mutation in gene DMD newer show traits/symptoms of disease 
b. In affected people, creatinine kinase (CK) levels are significantly increased 
c. DMD gene consists of 12 exons ​
d. None of above 
 
1. Clinical expression of myotonic dystrophy: 
a. Decreases with the number of repeats of trinucleotides 
b. Increase with the number of repeats of trinucleotides 
c. Depends on deletion of gene DMPK 
d. Depends on duplication of gene DMPK 
  
1. In Charcot Marie Tooth disease, which is X linked with dominant pattern of inheritance: 
a. We shouldn’t expect 2 times bigger amount of affected women than men 
b. All the women are asymptomatic carriers 
c. There are father to son inheritance patterns 
d. In  the  relationship  of  affected  male  with  not  affected  female  all  daughters  may  present 
symptoms of the disease 
  
1. Inherited  neoplasms  characterized  by  presence  of  germinal  mutation  of  suppressor  gene 
account  for  about  10%  of all the neoplasms. For one type of neoplasm this percentage can be 
lower,  for  others  significantly  higher.  From  the  examples  below,  choose  only  this  in  which 
the possibility of diagnosis of germinal mutation is above 10%: 
a. Lung cancer 
b. Ovarian cancer 
c. Cervical cancer 
d. Laryngeal cancer 
 
1. …  Graves’  disease  was  conducted  …  applied  for  the  tests  before  getting  a  job.  From  those 
people  11  were  diagnosed with the disease, 7 of those were carriers of risk genotype. Due to 
small number of diseased patients the group was enlarged up to 1100 people being signed in 
the clinic. All patients were screened and the results are below: 

 
 
Then RR was counted. RR equals 1,31. Assess the accuracy of this test: 
a. RR  was  correctly  counted,  but  based  on the research it is not a reliable test assessing influence 
of the genotype on the behavior ??? 
a. The  RR  value  was  not  correctly  counted, because it should be 1,75. Independently from 
counted  value,  RR  is  not  reliable  parameter  assessing  influence  of  genotype  on 
behavior based on conducted test. 
b. RR  was  correctly  counted  and  it  is  reliable  parameter,  assessing  the  influence  of  genotype  on 
the risk of being diseased based on conducted tests. 
c. RR  value  was  incorrectly  counted  and  it  is  1,75.  Independently  from  counted  value,  RR  is  a 
reliable  parameter  assessing  influence  of  the  genotype  on  the  risk  of  being  diseased  on  the 
basis of conducted tests. 
 
1. If  the  frequency  (allelic)  of  occurrence  of  allele,  which  causes  disease  inherited  in  the 
autosomal  recessive  pattern,  is  1/40  according  to  Hardy-Weinberg  equation,  frequency  of 
disease and carriers is approximately: 
a. 1/1600 and 1/20 
b. 1/800 and 1/20 
c. 1/1600 and 1/40 
d. 1/800 and 1/40 
 
1. Choose true sentence concerning aberrations of sex chromosomes: 
a. Males 47, XXY are fertile on condition they are given testosterone substitute 
b. The most frequent result of karyotype in women/girls with Turner syndrome is 45X/46XX 
c. Estrogen substitution in Turner syndrome prevent cardiovascular complications 
d. In  Klinefelter  syndrome  there  are  indications  for  prophylactic  gonadectomy  due  to  high  risk  of 
testicular cancer. 
 
1. Choose the false sentence describing Pierre Robin sequence: 
a. Is genetically heterogeneous; genetic factors account for 40-50% of its basis 
b. Pierre Robin's sequence is never genetically based/coded/inherited 
c. Significant  percentage  of  genetic  causes  of  sequences  are  micro  aberrations  detected  in  aCGH 
test, including DiGeorge syndrome and connective tissue diseases 
d. Micrognathia in sequence may result from intrauterine pressure (deformity) 
 
1. Choose the ​false​ sentence: 
a. Aneuploidy in autosomes is always lethal 
b. Trisomies are more frequent as a result of mistake in first meiotic division  
c. In case of trisomies, additional chromosome has most commonly maternal origin 
d. Trisomies in mosaic version are characterized by less severe course of disease 
 
1. Unbalanced aberration is described by the karyotype: 
a. 46,XY,t(4;7)(p14;q23) 
b. 46,XX,der(1)t(1,9)(qter;qter) 
c. 46,XY,inv(3)(p21q13) 
d. None of the above 
 
1. In  which  case  we  can  observe  the  highest  probability  of  spontaneous  abortion  during 
pregnancy: 
a. Down’s syndrome 
b. Neurofibromatosis type 1 
c. Huntington’s disease 
d. Trisomy of chromosome 18 
 
1. Choose the clinical symptoms in newborn, which suggest congenital metabolic defect: 
a. Seizures 
b. Lethargy or coma 
c. Difficulties in feeding and vomiting 
d. All of above 
 
1. Which  of  the  following  results  of  karyotype  testing  is  the  most  probable  in  phenotypically 
healthy woman who is infertile: 
a. 46,XX/45,X 
b. 47,XXX 
c. 46,XX 
d. 45,X 
 
1. 35  year  old  woman  with  muscle  weakness  and  receipt  hearing  loss.  Considering  possible 
metabolic causes, first of all we should measure: 
a. Ammonia in the serum 
b. Mucopolysaccharides in urine 
c. Lactic acid and alanine in the blood 
d. Neurotransmitters in CSF 
 
1. Typical  indication  for  Preimplantation  Genetic  Diagnosis  (PDG)  is  the  carriage  by  one  of  the 
spouses: 
a. Deletion of SMN1 gene (spinal muscular atrophy) 
b. Pathogenic variant in CFTR gene (cystic fibrosis) 
c. Balanced translocation 
d. All of above 
 
1. Parents,  whose  first child has an autosomal disease come to the clinic , because their second 
child  was  also  born  with  the  disease.  With  the  first  child  they  used  genetic  tests  (DNA 
isolated  from  the  blood)  and  it  appeared,  that  mutation  causing  the  disease  is  de  novo 
mutation  and  a  genetic  risk  in  the  next  child  is  low.  What is the probable explanation of this 
situation: 
a. This is the second time when the same de novo mutation occurred 
b. Mosaicism of germ line in one of the parent 
c. The reason of the disease is unidentified environmental factor 
d. None of above 
 
1. In  the  newborn,  whose  mother  has  uterine  leiomyoma,  asymmetrical  flattening  of  the  face 
and club feet were diagnosed. How would you classify those anomalies in the child: 
a. Malformation 
b. Deformation 
c. Dysplasia 
d. Rupture/discontinuance 
 
1. Single flexion line is frequent symptom in Down syndrome. It is an example of: 
a. Deformation 
b. Sequence 
c. Rupture/discontinuance 
d. Malformation 
 
1. Explain: 46,XY, der(19);t(4;19)(p15.3;q13.2)mat: 
a. Male  with  trisomy  of fragment of short arm of chromosome 4 and monosomy of fragment of 
long arm of chromosome 19 
b. Male  with  trisomy  of  fragment  of  short  arm  of  chromosome 19 and monosomy of fragment of 
long arm of chromosome 4 
c. Male  with  trisomy  of  fragment  of  long  arm  of  chromosome  19  and  monosomy  of  fragment  of 
short arm of chromosome 4 
d. Male  with  trisomy  of  fragment  of  long  arm  of  chromosome  4  and  monosomy  of  fragment  of 
short arm of chromosome 19 
 
1. The student had to count probability that healthy woman is a carrier of pathogenic allele. Her 
father  is  diseased  and  the  conducted  test  with  sensitivity  0.75  gave  a  negative  result.  The 
disease  is  inherited  in  autosomal  dominant  pattern,  homozygosity  is  lethal,  penetration  is 
0.8.  We  can  omit  de  novo  mutations.  Which  student  filled  the  table  properly?

 
a. Student A 
b. Student B 
c. Student C 
d. Student D 
 
1. In the micro-Warburg syndrome we can observe following clinical features: 
a. Microcephaly 
b. Microphthalmia 
c. Pseudohypertrophy of calf muscles 
d. Answers A and B are correct 
 
1. In which case the risk of recurrence of autosomal recessive disease is the highest: 
a. Lineage A 
b. Lineage B 
c. Lineage C 
d. Risk is the same in all lineages 
 
1. Choose the false sentence: 
a. Important  parameters  of  analysis  of  coupling  (??)  is  type  of  inheritance  and  theta,  which is the 
frequency of recombination between loci of marker and loci of the disease 
b. Lod score is customary form of presenting LR value in coupling analysis 
c. Probability  assuming  that  there  is  no  coupling  is  equal  with  probability  assuming  that  there  is 
coupling when theta is equal to zero. 
d. Coupling analysis is used in indirect diagnosis of the presence of unknown pathogenic mutation 
 
1. Which of the karyotypes indicates cytogenetic polymorphism in chromosome 9: 
a. 46, XY,dup(9q) 
b. 46, XY,9qh- 
c. 46,XY,del(9q) 
d. All of above 
 
1. Define  the  risk  of  occurrence  of  antigen  incompatibility,  which  can  result  in  serological 
conflict  in  the  case, when mother has phenotype Rh negative and father has genotype D/d in 
the gene: 
a. 25% 
b. 50% 
c. 75% 
d. 100% 
 
1. Patau syndrome is connected to the karyotype: 
a. 47, XX, +13 
b. 46, XX, der(13,14)(q10;q10) +13 
c. 47, XY, +13[10]/46,XY[10] 
d. All of above are correct 
 
1. Lyon hypothesis: 
a. Relationship factor 
b. Penetration depending on age 
c. De novo mutation 
d. Inactivation of chromosomes X 
 
1. Mark ​wrong​ sentence 
a. Germline mosaicism in parent decreases genetic risk in the offspring. 
b. People who are germline mosaics do not show the symptoms of the disease. 
c. In germline mosaicism mutation is present in the cells of sex differentiation line. 
d. Somatic cells of person, who is germline mosaic, are not affected by mutation. 
 
1. Choose the false sentence concerning coupling analysis: 
a. In  the  two  generation  family  the  phase  is  usually  unknown  and  calculations  are  made  for  two 
phases. 
b. In  the  three  generation  family  phase  can  be  determined,  so  the  calculations  are  made  only  for 
one phase. 
c. In the two generation family the phase is always known. 
d. For two families we always determine phases independently. 
 
1. Choose the false sentence concerning sequencing of the next generation: 
a. DNA  of  patients  is  prepared  according  to  special  protocol,  which  leads  to  creating  so  called 
“libraries”. 
b. In  the  process  of  sequencing  we  can  observe  migration  of  DNA strands in the gel following the 
rules of electrophoresis. 
c. Cluster  is  an  aggregation  of  DNA  strands  arising  in  the  process of bridge amplification (??) with 
one molecule of DNA. 
d. Many DNA molecules is sequenced at once. 
 
1. Penetration  of  the disease on the 80% level means that in the group of 100 people having the 
genotype causing the disease: 
a. 20 people will show symptoms of the disease 
b. 20 people will not present disease 
c. 80  people  will  manifest  fully  blown  disease,  in  20  people  disease  symptoms  are  going  to  be 
negligible 
d. 20  people  will  manifest  fully  blown  disease,  in  80  people  disease  symptoms  are  going  to  be 
negligible 
 
1. Choose the correct sentence: 
a. Crossing-over between loci on the same chromosome can lead to recombination. 
b. Loci  which  are  close  on  the  same  chromosome  undergo  recombination  in  less  than  50%  of 
cases, which means that they are coupled. 
c. The  distance  between  loci  can  be  expressed  in  centimorgans,  where  1  cM  corresponds  with 
frequency of recombination of about 1%. 
d. All of above 
 
1. Multifactorial  disease  occurs  with  frequency  of  1/100  in  males  and  1/300  in  females.  You 
are  analyzing  families  in  which  the  disease  appears  in  one  of  the  parents.  In  which  case  of 
the  following  combinations  parent-child  there  is  the  highest  risk  of  reappearance  of  the 
disease: 
a. Father - son 
b. Father - daughter 
c. Mother - son 
d. Mother – daughter 
 
1. Choose the proper genotype definition: 
a. Genotype is a mutation occurring in the investigating gene in the diseased patient 
b. Genotype is the whole human genetic material 
c. Genotype is unique system of alleles in particular locus/loci 
d. Genotype is unique system of alleles on only one chromosome 
 
1. Causes of genetically determined infertility: 
a. Aberrations in sex chromosomes 
b. Mutation of CFTR gene 
c. Permutation of FMR1 gene ( fragile X syndrome) 
d. All of above 
 
1. Neurofibromatosis type I is characterized by conditional mode: 
a. Autosomal recessive 
b. Autosomal dominant 
c. Recessive X linked 
d. Mitochondrial 
 
1. Phenotypic  trait  that  are  diagnostically  important  and  increase  the  likelihood  of 
identification of genetically determined cause of autism spectrum disturbances 
a. Macrocephaly 
b. Dysmorphia in stature 
c. Seizures/epilepsy 
d. All of above 
 

RANDOM 1  
1. Which of the following is inherited in an autosomal recessive condition? 
A. Achondroplasia 
B. Phenylketonuria 
C. Factor VIII deficiency (haemophilia A) 
D. Neurofibromatosis 
E. Spina bifida 
 
2.  A  36  year  old  woman  has  an  only  son  whom  haemophilia  is  diagnosed  ​.  ​Father  is  also  affected. 
Which of the following statements is correct?  
A. The condition is X-linked dominant 
B. All of her future daughters will be carriers 
C. All of her sons will be carriers 
D. Her son’s sons will be at 50% risk of haemophilia 
E. Fetal diagnosis of haemophilia for future pregnancy by DNA analysis is most used 
 
3.  A  man  and  a  woman  have  short  stature,  prominent  forehead,  shallow  nasal  bridge,  short  limbs 
and  other  skeletal  changes.  Both  have  been  told  that  they  have  achondroplasia  (100800), and wish 
to  know  their  recurrence  risk  for  affected  children.  Match  the  recurrence  risks  for  the  child  of 
achondroplastic parents to be affected (heterozygous and homozygous) 
A. 100% 
B. 75% 
C. 50% 
D. 25” 
E. Virtually 0 
 
4.  Following  the  birth of an affected child, the probability for the second child of the achondroplastic 
parents to be affected  
A. 100% 
B. 75% 
C. 50% 
D. 25% 
E. Virtually 0 
 
5. The cytogenic term “6q+” refers to 
A. 46, XX, dup (6q) 
B. Extra chromosome material derived from the long arm of chromosome 6 
C. 46, XX, dup (6) 
D. Extra chromosome material, origin unspecified, attached to the long arm of chromosome 6 
E. 47, XX, +6 
 
6. Mark true statement regarding family tree shown below 
 
 
 
 
A. I-2 is dead 
B. II-2 is a male 
C. III-3 is affected 
D. III-1 and III-3 are a consanguineous couple 
E. II-3 has one boy 
 
 
 
 
 
7. Mark false statement regarding family tree below 
 
 
 
 
A. The condition is X-linked  
B. II-5 is a non-penetrant gene carrier 
C. There is a 50% risk that any offspring of II-3 will carry the faulty 
D. The condition is likely to affect males and females equally 
E. II1-1 and III-2 have 50% of their genes in common 
 
8. Which of the following statements on birth defects is true 
A. Occur in 3:1000 live births 
B. In most children involve multiple organs 
C. Are common in chromosomal abnormalities 
D. Are almost all due to known single-gene defects 
E. Always involve malformations 
Questions 9-12: 
For each clinical description given below, select the most closely associated kind of genetic disease  
A. Chromosome abnormality 
B. Multifactorial condition 
C. X-linked disease 
D. Autosomal recessive disease 
E. Autosomal dominant disease  
 
9. Due to a combination of genetic and non genetic influences …………… 
 
10. A mendelian disorder characterized on the X chromosome……….. 
 
11.  Occurs  only  when  both  genes  in  a  pair  are  mutant and located on a chromosome other than X or 
Y………… 
 
12.  The  cause  of  most  common  diseases  of  adulthood  such  as  coronary  artery  disease  and 
non-insulin-dependent diabetes……… 
 
13. Which of the following conditions have polygenic inheritance? 
A. Galactosemia 
B. Polydactyly 
C. Meningomyelocele 
D. Turner syndrome 
E. Marfan syndrome 
 
14. What proportion of early spontaneous abortions are chromosomal? 
A. 1% 
B. 5% 
C. 10% 
D. 25% 
E. 50% 
 
15. Anticipation is characteristic of conditions caused by 
A. Microdeletions 
B. Mitochondrial inheritance 
C. Genomic imprinting 
D. Trinucleotide repeat expansions 
E. Germline mosaicism  
 
 

RANDOM 2 
1. Which would NOT be an effective way to diagnose sickle cell anemia?  
A. PCR exon 1 of the beta-globin gene, followed by restriction digest with MstII 
B. Hemoglobin electrophoresis 
C. Restriction  digest  of  genomic  DNA  with  MstII,  followed  by  southern  blot,  using a beta-globulin 
gene product 
D. Enzymatic assay for hemoglobin activity 
 
 
2. A human male carrying an allele for a trait on the X chromosome is 
A. Heterozygous 
B. Homozygous 
C. Hemizygous 
D. Monozygous 
 
3. Mark the sentence that is false for chromosome X inactivation 
A. Inactivation occurs in somatic and germ line cells 
B. Occurs early in embryonic life 
C. Is random  
D. Inactivate chromosome is present as a Barr body 
 
4.  Which  of  the  following  procedures  is  used  as  a  routine  technique  for  karyotyping  using  light 
microscopy 
A. C-banding 
B. Fluorescence in situ hybridization 
C. G-banding 
D. Q-banding 
 
5. If a person with blood type AB mates with someone with blood type 0, what are the possible blood 
types of their offspring 
A. AB, 0 
B. A, B, 0 
C. A, B 
D. A, B, AB, 0 
 
6. Haemolytic disease of newborn (HDN) is a result of 
A. Rh factor conflict between Rh(-) mother and Rh(+) child 
B. Rh factor conflict between Rh(+) mother and Rh(-) child 
C. Being mother affected with sickle cell anemia 
D. Being mother affected with malaria during pregnancy 
 
7.  For  a  given  autosomal  recessive  disease,  q=0,02  (where  q  is  the  allele  frequency  of  the  mutant 
allele). Approximately what percentage of the population has two copies of the normal allele? 
A. 19% 
B. 1,9% 
C. 96% 
D. 98% 
 
8. Mark the false sentence describing mitochondrial inheritance  
A. Only affected females transmit the disease to offspring 
B. Neuropathies and myopathies are the most common mitochondrial diseases 
C. Mitochondrial DNA particles are circular single stranded 
D. Different cells vary in number of mt DNA copies  
 
9. Late paternal age can result in higher frequency of 
A. Genomic imprinting 
B. New mutations 
C. Mitochondrial mutations 
D. Chromosomal aberrations  
 
10. Mark the disease which is N​OT​ caused by expanded triplet repeats 
A. Myotonic dystrophy 
B. Duchenne muscular dystrophy 
C. Huntington disease 
D. Fragile X syndrome 
 
11. Rett syndrome, lethal in hemizygous males, is an example of 
A. Dominant x-linked inheritance 
B. Recessive x-linked inheritance 
C. Co-dominant inheritance 
D. Mitochondrial inheritance  
 
12. Mark the false sentence describing fragile X syndrome 
A. Is one of the most common causes of mental retardation in males 
B. The frequency is 1 in about 2000 male live births 
C. Can be detected using banding techniques and molecular methods 
D. Is dominant X-linked disease 
 
13.  Which  of  the  following  disorders,  except  Tay-Sachs  disease,  is  also  more  common in Ashkenazi 
Jews 
A. Gaucher disease 
B. Sickle cell anemia 
C. Cystic fibrosis 
D. Niemann-Pick disease 
 
14. Which of the following is NOT a chromosomal aberration 
A. Frame shift 
B. Deletion 
C. Translocation 
D. Inversion 
 
15. Which disorders are examples of uniparental disomy 
A. Prader-Willi and Angelman syndrome 
B. Duchenne and Becker muscular dystrophy  
C. Alpha and beta thalassemia 
D. Trisomy 21 and 22 
 
16. The karyotype 46,XX del Xp can occur in females with 
A. Testicular feminization syndrome 
B. Turner syndrome 
C. “Superwoman” syndrome  
D. None of these 
 
17. Which statement about down syndrome is false 
A. The frequency increases is mothers over age 35 
B. The cause is a nondisjunction 
C. The only aberration is a simple trisomy of 21 pair 
D. There are patients with translocation i.e. t(14;21) 
 
18. Match the karyotypes with the probable phenotypes 
1. Fragile X syndrome 
2. 45, XX 
3. Unbalanced translocation carrier 
 
a. Neonatal death 
b. Reduced fertility 
c. Mental retardation 
 
A. 1a, 2c, 3b 
B. 1c, 2a, 3b 
C. 1a, 2b, 3c 
D. 1b, 2a, 3c  
 
19. The acrocentric chromosomes are 
A. 21, 22, and Y 
B. 13, 14 and 15 
C. 13, 14, 15, 21 and 22 
D. X and Y 
 
20. Karyotyping in prenatal diagnosis is important in cases of 
A. Pregnancy of females who had miscarriages 
B. Pregnant females that already have 1 child with congenital defects 
C. Pregnant females after age 36 
D. All above  
 
21. The banding techniques are used for detection of 
A. All structural aberrations 
B. Number and big structural aberrations 
C. Microdeletions 
D. Aneuploidies 
 
22. FISH as a technique for mutation diagnosis in human genetics is essential for detection of: 
A. Angelman syndrome 
B. Turner syndrome 
C. XYY males 
D. Cri du chat syndrome 
 
23. CML is caused by translocation between abl and bcr genes which determines male karyotype: 
A. 46, XY, t(9;21) 
B. 46, XY, t(9;22) 
C. 46, XY, t(21;22) 
D. 46, XY, t(11;22) 
 
24. Many different mutations within CFTR gene cause 
A. Galactosemia 
B. Tay-Sachs disease 
C. Cystic fibrosis 
D. Albinism  
 
25. Fluorescently labelled molecular probes are used in 
A. PCR 
B. FISH 
C. AFLP 
D. SSCP 
 
26. PCR-RFLP can be useful in  
A. Related species identification 
B. Point mutation detection 
C. Population studies 
D. All answers are correct 
 
27. VNTR sequences are mostly used in 
A. Point mutations determination 
B. Detection of pathogens 
C. Forensic medicine 
D. Chromosome aberration study 
 
28. The frequency of phenylketonuria is 1 per 7000 (in Europe). What is the frequency of carriers? 
A. 1,195% 
B. 0,988% 
C. 2,361% 
D. 1,180% 
 
29.  All  of  the  following  karyotypes  are  found  in  spontaneous  abortuses.  Which  of  the  following  is 
least likely to be found in a live-born infant? 
A. 45, X 
B. 47, XX, + 21 
C. 47, XX, + 16 
D. 47, XY, +18 
 
30. Which of the following syndromes is associated with maternal disomy of chromosome 15? 
A. Prader Willi syndrome 
B. Angelman syndrome 
C. Klinefelter's syndrome 
D. Rett syndrome  
 
31. Fragile X syndrome is associated with 
A. Chromosome 16 abnormalities 
B. A chromosomal deletion at Xq28 
C. Expansion of CAG triplet repeat 
D. Expansion of CGG triplet repeat 
 
32.  A  nineteen  year  old  female  with  short  stature,  wide  spaced  nipples,  and  primary  amenorrhea 
most likely has the karyotype of 
A. 47, XXX 
B. 47, XXY 
C. 45, X 
D. 47, XX, + 18 
 
33. The majority of cases of Down syndrome occur as a result of 
A. Nondisjunction during paternal meiosis 
B. End to end fusion of two chromosomes 21 
C. Nondisjunction during maternal meiosis 
D. Mosaicism of normal and trisomic cell lines  
 
34. A person with two or more different cell lines originating from a single zygote is called a 
A. Hemizygote 
B. Chimaera 
C. Mosaic 
D. Carrier 
 
35. Technique useful for person identification is 
A. DNA fingerprint analysis 
B. Northern blot 
C. SSCP 
D. RFLP analysis 
 
36. Nucleic acid amplification is performed in vitro by: 
A. PCR and southern 
B. Southern and northern 
C. PCR and RT-PCR 
D. RT-PCR and northern 
 
37. It is not possible to detect frameshift mutation by  
A. PCR and RT-PCR 
B. Western (immunological) techniques 
C. Southern techniques 
D. Northern techniques 
 
38. Molecular techniques are useful for 
A. Point mutation detection 
B. gene(s) expression analysis 
C. Parasites detection and determination 
D. All above 
 
39. For eukaryotic gene(s) expression in Escherichia coli it is obligatory to use 
A. cDNA and expression vector 
B. Genomic DNA and expression vector 
C. cDNA and YAC vector 
D. Genomic DNA and YAC vector 
 
40. Molecular assay for paternity determination can involve 
A. Mitochondrial DNA polymorphism analysis 
B. Ribosomal DNA polymorphism analysis 
C. Microsatellite DNA polymorphism analysis 
D. Promoter polymorphism analysis  
 
41. What kind of inheritance is the pedigree showing 
 
 
 

 
 
 
A. Mitochondrial 
B. X-linked recessive    
C. Autosomal dominant 
D. X-linked dominant 
 
42. The pedigree pattern is an example of 
 
 

 
 
 
A. Autosomal dominant inheritance 
B. Autosomal recessive inheritance 
C. X-linked recessive inheritance 
D. X-linked dominant inheritance 
 
MCQ’s FROM BOOK  
 
Select the best single answer to the following questions: 
 
1.  A  term  female  infant  to  a 37-year-old mother with three prior children has a low birth weight and 
a  poor  latch  for  breast-feeding  the  first  24  hours  of  life.  Mother had first trimester maternal serum 
screening  (quad  screen)  that  was  normal.  Your  assessment  of  the  baby  reveals  an  unusual  facial 
appearance  with  a  broad  nose  and  extra  skin  folds  on  the  neck.  Based  on  the  history,  which  of  the 
following is the most likely reason for poor breast-feeding in this child: 
A. Maternal incompetence 
B. Autosomal dominant disorder in mother 
C. X-linked recessive disorder in child 
D. Chromosomal disorder in child 
E. Multifactorial disorder in child 
 
2.  Prior  to  receiving  test  results,  the  most  important  aspect  of  care  along  with  evaluating  the 
feeding problem is: 
A. Genetic counseling regarding recurrence risk 
B. Genetic counseling regarding prenatal diagnosis 
C. Supportive counseling for future mental retardation 
D. Supportive counseling for probable birth defects 
E. Supportive counseling explaining the management plan  
 
3.  A  female  infant  demonstrates  inconsistent  bottle  feeding  and  exaggerated  jaundice  with  a  total 
bilirubin  of  14  at  day  2  of  life.  Your  assessment  reveals  the  infant  is  less  responsive  than  early  on 
your  shift,  and  you  note  decreased  muscle  tone  with  a  poor  suck.  The  prenatal  history  is  normal 
except  that  the  mother  and  father are from Pakistan and are second cousins. Which of the following 
conditions would be most likely in this infant? 
A. Chromosome disorder 
B. Biliary atresia 
C. Inborn error of metabolism 
D. Lactose intolerance 
E. Multifactorial disorder 
 
4.  As  the  infant  is  being  evaluated,  a  grandparent  brings  documentation  from  Pakistan  showing  a 
prior  child  of  this  couple  died  with  a  diagnosis  of  maple syrup urine disease. Which of the following 
would resources would provide information on the inheritance of this disorder? 
A. Online Mendelian Inheritance in Man 
B. Gene Tests 
C. Alliance of Genetic Support Groups 
D. ISONG 
E. ACOG 
 
5.  A  21-year-old  female  was  referred  to  obstetric  clinic  from  the  emergency  room after a diagnosis 
of  malnutrition  and  a  positive  pregnancy  test.  She  had  been  brought  in  by  the  police  for  vagrancy 
and  alcoholism,  exhibiting  poor  hygiene  and  nutrition  on  examination.  She  also  was  affected  with 
cystic  fibrosis,  having  a  milder  disease  course,  and  a  sister  had  a  child  with  spina  bifida.  Fetal 
ultrasound  revealed  a  fetus  of  about  3  months  gestation  with  very  small  head  circumference, 
abnormal  head  shape,  and  intrauterine  growth  retardation.  The  poor  malnutrition  and  unplanned 
pregnancy caused the young woman to miss the following standards of care: 
A. Amniocentesis because of higher risks for chromosome abnormalities and cystic fibrosis 
B. Triple/Quad screening with ultrasound to screen for fetal chromosome abnormalities 
C. Preconception counsel including provision of vitamins with folic acid 
D. Prosecution because of suspected alcoholism causing damage to the fetus 
E. Preimplantation genetic diagnosis of to avoid the high risk for fetal cystic fibrosis 
 
6. Which of the following birth defects would be most likely to occur in this situation? 
A. Congenital heart defect 
B. Omphalocele 
C. Anencephaly 
D. Tracheo-esophageal fistula 
E. Anal atresia 
 
7.  A  Caucasian  couple  in  the  20s  comes  in  for  preconception  counseling  regarding  their  first 
pregnancy.  They  have  had  no  prior  miscarriages  or  infertility  and  their  family  histories  are  normal. 
This  lack  of  risk  factors  means  that  their  risk  for  fetal  abnormalities  in  this  pregnancy  is 
approximately: 
A. 50%  
B. 25%  
C. 10% 
D. 2-3% 
E. <1% 
 
8. Which of the following genetic screening tests should be considered for this couple? 
A. Alpha-thalassemia 
B. Beta-thalassemia 
C. Tay-Sachs disease 
D. Sickle cell anemia 
E. Cystic fibrosis 
 
9.  A  couple  present  to  an  obstetric  nurse  for  counseling  because  they  have  had  three  early 
miscarriages  at  6-8  weeks  gestation.  Both  are  in  good  health  without chronic illnesses, and neither 
has  any  family  history  of  birth  defects  or  miscarriages.  Which  of  the  following  is  an  important 
contributor to miscarriages that can be tested in this couple? 
A. Autosomal dominant disorders 
B. Chromosomal disorders 
C. Multifactorial disorders 
D. Mitochondrial disorders 
E. X-linked recessive disorders 
 
10.  Which  of  the  following  results  is  most  plausible  for  this  couple,  along  with  its  likelihood  given 
their history? 
A. Trisomy,1% 
B. Trisomy,10% 
C. Translocation,2-3% 
D. D. Translocation 20-30%  
E. E. Turner syndrome,10% 
 
11.  A  woman  is  diagnosed  with  Crohn’s  disease,  and  wishes  to  know the risk that her daughter will 
develop  the  disease.  She  is  otherwise  normal  with  an  unremarkable  family  history.  The  likely 
inheritance mechanism and her daughter’s risk would be: 
A. Autosomal dominant with a 50% risk 
B. Autosomal recessive with a 25% risk 
C. X-linked recessive with a 25% risk 
D. Chromosomal with a 10-15% risk 
E. Multifactorial determination with a 5-7% risk 
 
12.  A  24-year-old  Ashkenazi  Jewish  woman  develops  bilateral  breast  cancer.  Her  mother  and 
grandmother died of ovarian cancer, and a maternal aunt also had early onset breast cancer. She has 
two daughters aged 12 and 16. The most probable mechanism and risk to her daughters would be: 
A. Multifactorial determination with a 1-2% risk 
B. Autosomal dominant with a 50% risk 
C. Autosomal dominant with a 25% risk 
D. X-linked dominant with a 50% risk 
E. X-linked dominant with a 25% risk 
 
13.  (pectus),  long  fingers,  flat  feet,  and  increased  joint  laxity.  His  father  died  at age 35 with a heart 
attack.  He  wants  approval  to  play  basketball.  An  important  disease  category  and  disorder  to 
consider would be: 
A. Coronary artery disease and myocardial infarction 
B. Coronary artery disease and congestive heart failure 
C. Connective tissue disease and aortic dilatation 
D. Connective tissue disease and myocardial infarction 
E. Connective tissue disease and aortic coarctation 
 
14.  A  30-year-old  man  has  hypertension  controlled  by  diet  and  medication,  and  one  of  his  three 
siblings  is  affected.  His  father  died  of  kidney  failure,  and  one  of  the  man’s  three  sons  had  urinary 
tract infections with cystic kidneys on ultrasound. The most likely diagnosis is: 
A. Multifactorial predisposition to renal failure 
B. Isolated congenital anomaly of the urinary tract 
C. Autosomal dominant polycystic kidney disease 
D. Autosomal recessive polycystic kidney disease 
E. X-linked recessive polycystic kidney disease 
 
15.  At her first obstetric visit, a woman tells you she has a brother with mental retardation. She asks 
what the risk for mental retardation will be for her current pregnancy. You reply: 
A. Mental retardation is a complex phenotype that is rarely genetic. 
B. Mental retardation fits into the polygenic category with a low recurrence risk. 
C. It is imperative to establish a more specific diagnosis before counseling can be provided. 
D. It is imperative to perform a karyotype on her brother before counseling can be provided. 
E. The risk is significant but there is no prenatal diagnosis for mental retardation. 
 
16. An individual with genotype Aa at a genetic locus will produce: 
A. Only gametes with genotype Aa 
B. Only gametes with genotype a 
C. Only gametes with genotype aa 
D. Half of gametes with genotype A and half with genotype a 
E. Only gametes with genotype AA 
 
17.  If  allele  B  causes  disease  and  allele  b is associated with a normal phenotype, what is the chance 
that  a  baby  born  to  a  Bb  mother  will  have  the  disease?  Assume  that  the  father  has  a  bb  genotype 
and that there is no variable expressivity. 
A. 100% 
B. 75% 
C. 50% 
D. 25% 
E. Less than 1% 
 
18.  The  allele  for normal hemoglobin is represented as A, and that for sickle hemoglobin as S. A man 
with  sickle  cell  trait  (genotype  AS)  marries  a  woman  who  is also sickle trait. What are their chances 
to have a child with sickle cell anemia (genotype SS)? 
A. 100% 
B. 75% 
C. 50% 
D. 25% 
E. Less than 1% 
 
Match the following questions with one or more correct answers: 
 
A  man  learns  that  he  has  an  abnormal  allele  Z  for a particular type of liver disease that is extremely 
rare  in  the  general  population.  If  his  genotype is Zz, what are the chances for the following relatives 
to have the Z allele? Assume that no mutations have occurred. 
A. 3/4 
B. 1⁄2 5 
C. 1⁄4 7 
D. ⅛ 6 
E. 1/16 
 
2-5. His son: B 
2-6. His first cousin: D 
2-7 his grandson: C 
 

 
3.1.  An  affected  male  infant  born  to  normal  parents  could  be  an  example  of  all  the  following 
disorders except: 
A. an autosomal dominant disorder 
B. an autosomal recessive disorder 
C. a polygenic disorder 
D. a vertically transmitted disorder 
E. an X-linked recessive disorder 
 
4.1.A  father  has  a  child  with  a  cleft palate and learns that his mother had a heart defect. He has two 
normal children. What is the risk for his next child to be have cleft palate? 
A. 35-50% 
B. 6-8% 
C. 3-5% 
D. 0.7% 
E. 0.2% 
 
4.2.  A  father  with  unilateral  cleft  lip/palate  has  a son with unilateral cleft lip/palate. His risk for the 
next child to be affected with unilateral cleft lip/palate is 
A. 38-40% 
B. 10-12% 
C. 6-10% 
D. 3-4% 
E. 0.1% 
 
4.3.  Males  are  five-fold  more  likely  than females to have pyloric stenosis. Using this knowledge and 
figures  in  Table  4.1,  what  would be the recurrence risk for offspring of a father affected with pyloric 
stenosis 
A. 55-60% 
B. > 7.5% 
C. 7.5% 
D. < 7.5% 
E. 0.4% 
 
4.4.  A  mother  with  history  of  pyloric  stenosis  has which of the following risks that her newborn son 
will develop pyloric stenosis. 
A. 20.5% 
B. 13.2% 
C. 11.1% 
D. 6.8% 
E. 2.5% 
 
4.5.  A  46-year-old  man  hears  that  his  identical  twin,  separated  at  birth,  died  of  coronary  artery 
disease in the past year. What is his risk to develop the disorder? 
a. Higher than 39-44% 
b. 39-44% 
c. Lower than 39-44%, but higher than 30%  
d. 14-25% 
e. Lower than 14-15% 
 
4.6-4.9  Many  of  the  more  common  birth  defects  like  cleft  palate or congenital heart disease exhibit 
multifactorial  determination.  Although  specific  empiric  risks  can  be  specified  as  in  Tables  4.2  and 
4.3,  general  risks  can  be  borne  in  mind  relative  to  an  affected  person:  identical  twin,  20-30%; 
first-degree  relative,  3-4%;  two  first-degree  relatives,  5-8%;  three  first-degree  relatives,  9-12%; 
second-degree  relatives,  0.7-2  %,  third-degree  relatives  and  general  population,  less  than  0.5%.  By 
reference  to  the  person with a birth defect, match the relatives below with their proportion of genes 
in common and their concordance or recurrence risk: 
A. 100% genes in common, 20-30% concordance risk 
B. 50% genes in common, 3-4% concordance risk  
C. 50% genes in common, 3-4% recurrence risk  
D. 25% genes in common, 2% recurrence risk  
E. 12.5% genes in common, <0.5% recurrence risk 
 
4.6. Twin brother whose twin sister has cleft palate by ultrasound: ​B 
4.7. Unborn sibling of a child with congenital heart defect:​ ​C 
4.8. Grandchild of a person with spina bifida: ​D 
4.10  A  man  learns  that  his  fraternal  twin  has  developed  type  I  diabetes  mellitus  at  age  17.  He  has 
HLA  testing and finds that his DR haplotypes are different from his twin’s, and that he does not have 
the DR3 or DR4 allele. His risk to develop type I diabetes is 
A. 90% 
B. 30-50% 
C. 10-15% 
D. 4-6% 
E. < 4-6% 
 
5.1E 5.2E 5.3C 5.4E 5.5E 5.6B 5.7C 5.8C 
 
5.1  A  woman  has  a  brother  with  sickle  cell  anemia.  She  marries  an  unrelated  man  with  a  normal 
family  history.  Given  a  1/16  frequency  for  the  sickle  globin  allele,  what  is  their  risk  to  have  an 
affected child (equate 15/16 to 1)? 
(A) ¼ 
(B) 1/6  
(C) 1/8  
(D) 1/24 
(E) 1/48  
 
5.2  A  population has a 1/3000 frequency of individuals with cystic fibrosis. If all affected individuals 
were sterilized, the frequency of those affected in the next generation would be 
a) Reduced by 2/3 
b) Reduced by 1/2 
c) Reduced by 1/3 
d) Reduced to O 
e) Approximately the same 
 
5.3  In  a  population  in  which  random  mating  occurs,  50%  of  the  people  are  heterozygotes  for  a 
mutant allele. In the following generation, the frequency of heterozygotes will be: 
a) 100% 
b) 75% 
c) 50% 
d) 25% 
e) 0% 
 
5.4  Of  the  following  assumptions  that  must  be  valid  for  a  population  to  be  in  Hardy-Weinberg 
equilibrium, which is the least likely to hold for modern Americans? 
a) no positive or negative selection 
b) no assortative mating 
c) low rates of mutation 
d) large populations to dilute the effect of genetic drift 
e) no migrations to alter frequencies by gene flow 
 

 
5.5-5.7 Match each of the ethnic groups listed below with the genetic disorder commonly associated 
with it. 
a) Cystic fibrosis 
b) Alfa-thalassemia  
c) Tay sachs disease  
d) Alfa-1 antitrypsin deficiency 
e) Glucose-6-phosphate dehydrogenase deficiency  
 
5.5 african americans= ​E 
5.6 chinese americans =​B  
5.7 jewish americans = ​C 
 
A  man  loses  a  child  because  of  failure  to  recover  from  anesthesia.  A  diagnosis  of  succinylcholine 
apnea  is  made,  and  the  man  and  his  wife  test  positive  as  carriers  for  the  A mutant allele (see Table 
5.3).  Later,  the  man  remarries  and  has  a second family. What are the risks that his new children will 
be AA homozygotes?  
a) 1/30 
b) 1/60 6 
c) 1/240 
d) 1/480 
e) 1/540 
 
6.1  A  hypothetical  gene  contains  two  exons  that  encode  a  protein  of  100  amino  acids.  They  are 
separated  by  an intron of 100 bp beginning after the codon for amino acid 10. The gene’s messenger 
RNA (mRNA) has 5’ and 3’ untranslated regions of 70  
and  30  nucleotides,  respectively.  Match  the  characteristics  of  the  gene  with  the  appropriate 
measures below.  
(A) 500 bp 1 
(B) 400 bp 2 
(C) 300 bp 
(D) 100 bp 
(E) 70 bp 
 
6.1. Size of nuclear RNA transcript minus added poly(A) tail = ​A 
6.2. Size of cDNA made from mature mRNA= ​B 
 
6.3  to  6.5.  Recall  that  adult  hemoglobin  synthesis  requires  transcription  factors  that  activate  the  - 
and  -globin  genes,  splicing  to  form  mature  mRNA,  balanced  globin  protein  synthesis,  and 
association  of  the  globins  with  heme  to  form  a  functional  hemoglobin  molecule.  Match  the 
phenotypes below with the process(es) affected by mutation: 
a. Severe  anemia  with  targeted  cells  indicative  of  imbalanced  globin  protein  synthesis  Mild 
anemia with no cyanosis  
b. Mild anemia with obvious cyanosis 
c. Mild anemia with small, rounded red cells 
d. Severe anemia with vascular blockage causing heart attacks and strokes 
 
6.3. Deletion in the upstream promoter of the beta-globin gene = ​A 
6.4 Deletion of a 3 bp codon in an alfa-globin gene = ​B 
6.5 Frameshift mutation in the beta-globin gene= ​A 
 
6.6  to  6.10.  The  various  types  of  genetic  anemias,  like  many other genetic disease categories, must 
be  diagnosed  by  particular  laboratory  studies.  Match  the  following  alterations  of  -globin  gene 
expression with the test that will be sufficiently abnormal to allow diagnosis. 
(A). Point mutation upstream of the -globin gene causing mild thalassemia 9 
(B).  Deletion  of  the  second  translated  region  in  the  -globin  gene  (second)  exon to cause severe 
thalassemia 8 
(C). RNA splice site mutation to cause severe thalassemia 10 
(D). Amino acid substitution (charged to neutral) to produce an unstable -globin peptide 6 
(E). Amino acid substitution (neutral to neutral) in the binding site for heme. 7  
 
6.6. Analysis to detect altered mobility of beta-globin protein by electrophoresis = ​D 
6.7. Analysis to detect decreased oxygen binding by red blood cells=​ E 
6.8. Analysis to detect altered size of beta-globin protein = ​B 
6.9. Analysis to detect altered DNA sequence of promoter regions = ​A 
6.10. Analysis to detect altered size of beta-globin mRNA= ​C 
 
6.11. Which characteristic best refers to single nucleotide substitutions in human DNA?  
(A) They are rarely seen in introns 
(B) They usually result in disease 
(C) They may create a restriction fragment length polymorphism (RFLP) 
(D) They may create a VNTR allele  
(E) They may delete a codon 
 
6.12.  Which  of  the  following  methods  could  not  be  used  to  detect  the  point mutation in the -globin 
gene that causes sickle cell anemia? 
(A) Polymerase chain reaction (PCR) with allele-specific oligonucleotide (ASO) hybridization 
(B) Southern blot analysis 
(C) DNA sequencing 
(D) Polymerase chain reaction (PCR) with restriction enzyme digestion  
(E) Northern blot analysis 
 
7.1. The cytogenetic term “6q+” refers to 
(A) 46,XX,dup(6q) 
(B) extra chromosome material derived from the long arm of chromosome 6 
(C) 46,XX,dup(6p) 
(D)  extra  chromosome  material,  origin  unspecified,  attached to the long arm of chromosome 

(E) 47,XX,+6 
 
7.2–7.4. Match each clinical situation below with the appropriate risk figure. 
(A) 1/10,000 
(B) 1/800  
(C) 1/100  
(D) 1/10 
(E) 1  
 
7.2. The risk for a newborn to have Down syndrome = ​ 1/800 
7.3. The theoretical risk for a 21/21 translocation carrier to have a child with Down syndrome= ​1  
7.4.  The  risk  for  parents  of  a  trisomy  21  child  to  have  a  second  offspring  with  a  chromosomal 
abnormality = ​1/100  
 
7.5–7.7. Match each of the genetic conditions below with the correct cytogenetic notation. 
A. 47,XX,+21   
B. 45,X  
C. 47,XXX 
D. 47,XY,+21 
E. 45,XX,-21  
 
7.5. Male with trisomy 21 (Down syndrome) = ​47, XX, + 21 ; Male with trisomy 21 (Down Syndrome) 
7.6. Female with monosomy X (Turner syndrome)=​45, X ; Female with trisomy X (Turner Syndrome) 
7.7. Female with monosomy 21= ​45, XX, -21 ; Female with monosomy 21  
 
8.1 The discovery of genetic diseases due to expanding triplet repeats validated the concept of: 
A. Variable expression  
B. New mutation 
C. Genomic imprinting 
D. Anticipation 
E. Mitochondrial inheritance 
 
8.2  A  child  has  symptoms  of  neurofibromatosis-1  (NF-1;  162200),  including  café-au-lait  spots  and 
neurofibromas,  but  they  are  confined  to  his  right  leg  and  inguinal  region.  The  family  history  is 
normal. This is most consistent with: 
A. Mitochondrial disease 
B. Triplet repeat disease 
C. Gene expression influenced by parent of origin (imprinting effects) 
D. Somatic mosaicism 
E. Germline mosaicism 
 
8.3  In  a  large  family,  disease  A  occurs  in  offspring  when  their  mother  is  affected,  while  disease  B 
occurs in offspring when their father is affected. This pattern is suggestive of: 
A. Mitochondrial disease 
B. Triplet repeat disease 
C. Gene expression influenced by parent of origin (imprinting effects) 
D. Somatic mosaicism 
E. Germline mosaicism 
 
8.4  A  DNA  methylation  analysis  is  performed  on  a  gene  region  that  is  known  to  be imprinted in the 
mouse.  The  finding  of  identical  DNA  methylation  patterns  on  alleles  from  both  homologous 
chromosomes would occur in which tissue? 
A. Male gametes 
B. Heart tissue in adult females 
C. Liver tissue in adult males 
D. Male primordial germ cells 
E. Female neural crest cells 
 
8.5  Atypical  inheritance  mechanisms  have been revealed by molecular genetic technology, including 
all of the following except: 
A. sporadic mutation 
B. uniparental disomy and genomic imprinting 
C. mitochondrial inheritance 
D. trinucleotide repeat amplification 
E. maternal inheritance 
 
8.6  Normal  grandparents  have  a  grandson  with  the  fragile  X  syndrome.  Their  daughter  has  had 
learning differences in school, but is otherwise normal. The grandfather is most likely: 
a) An affected male with incomplete penetrance 
b) A transmitting male 
c) A son of a fragile X carrier 
d) Not the real father of his daughter 
e) Mosaic in his germline tissue 
 
8.7 to Match the following diseases with the genetic mechanisms indicated below: 
A. Huntington chorea A - ​triplet repeat expansion during female meiosis  
B. Prader-Willi syndrome 
C. Angelman syndrome 
D. Myotonic dystrophy 7 - ​triplet repeat expansion during male meiosis  
E. Hypomelanosis of Ito 
 
9.1.  Monozygotic  twins  with  connected  placental  circulations  can  develop  a  pattern  of  vascular 
occlusions  due  to  blood  clots.  A  twin  with a brain cyst, absent kidney, cleft palate, and absent digits 
has: 
A. a malformation syndrome 
B. a deformation syndrome 
C. a disruption syndrome 
D. a dysplasia syndrome 
E. anassociation 
 
9.2.  A  child  has  a  small  head,  minor  anomalies  of  the  face  including  a  thin  upper  lip,  growth  delay, 
and  developmental  disability.  His  mother  becomes  defensive  when asked about her pregnancy. The 
child is likely to have: 
A. a chromosomal syndrome 
B. a teratogenic syndrome 
C. a Mendelian syndrome 
D. a polygenic syndrome 
E. Anassociation 
 
9.3.  A  newborn  has  multiple  blood  vessel  tumors  (hemangiomas)  over  his  trunk  and  legs,  together 
with a large head and body asymmetry. He has: 
A. a malformation syndrome 
B. a deformation syndrome 
C. a disruption syndrome 
D. a dysplasia syndrome 
E. anassociation 
 
9.4.  A  child  has  congenital  contractures  (bent  limbs)  due  to  a  large  uterine  fibroid  that  her  mother 
developed during pregnancy. The child has: 
a. a malformation syndrome 
b. a deformation syndrome 
c. a disruption syndrome 
d. a dysplasia syndrome 
e. anassociation 
 
9.5.  Several  patients  are  described  with  similar  congenital anomalies including defects of the sacral 
vertebrae,  kidneys,  anus,  and  trachea.  The  children  do  not  have  minor  anomalies and their faces do 
not resemble one another. The child most likely has: 
a) a malformation sequence 
b) a deformation syndrome 
c) a disruption syndrome 
d) a dysplasia syndrome 
e) An association 
 
9.6-9.7.  A  family  requests  genetic  counseling  because  the  father  has widely spaced eyes, increased 
facial  hair  and  deafness.  One  of  their  three  children  also  has  deafness  with  similar  facial  features. 
The  mother  is  normal,  but  a  paternal  sister  and  the  paternal  grandfather  are  known  to  have 
deafness and increased facial hair. 
 
9.6. The most likely pattern of inheritance of this disorder is 
a) Autosomal dominant 
b) Autosomal recessive 
c) X-linked dominant 
d) X-linked recessive 
e) None of the above 
 
9.7 the recurrence risk for this couple is 
a) 100% 
b) 67% 
c) 50% 
d) 25% 
e) Virtually 0 
 
9.8-9.10.  A  man  and  woman  have  short  stature,  prominent  forehead,  shallow  nasal  bridge,  short 
limbs,  and  other  skeletal changes. Both have been told that they have achondroplasia (100800), and 
wish  to  know  their  recurrence  risks  for  affected  children.  Match  the  recurrence  risks  with  the 
questions below. 
(A) 100% 
(B) 75% - 8* 9* 
(C) 50% - 10* 
(D) 25% 
(E) Virtually 0 
 
8*  The  probability  for  the  first  child  of  achondroplastic  parents  to  be  affected  (heterozygous  or 
homozygous) 
9*.  Following  the  birth  of  an  affected  child,  the  probability  for  the  second  child  of  achondroplastic 
parents to be affected 
10*  The  probability  for  the  first  child  to  be  affected  if  an  achondroplast  marries  an  unaffected 
person 
 
10.  A  3-day-old  infant  is  the  fourth  child  of  immigrant  parents.  She  stops  feeding,  becomes 
lethargic,  and  appears  more  noticeably  jaundiced.  In  addition  to  cultures  of  blood,  cerebrospinal 
fluid,  and  urine,  a  blood  pH  of  7.1  is  found.  The  electrolytes  show  a  sodium  of  135,  potassium  4, 
chloride  99,  and  bicarbonate  15  milliequivalents  per  liter.  A  family  history  reveals  that  the  parents 
are first cousins, and that prior child died in the nursery because of infection. 
 
10.1. The blood pH and electrolyte values suggest:  
A. Alkalosis with high bicarbonate 
B. iÍncreased anion gap with acidosis 
C. Increased anion gap with alkalosis 
D. Acidosis with high bicarbonate 
E. Alkalosis with low bicarbonate 
 
10.2. The most important metabolic test in this setting is: 
A. Blood glucose 
B. Blood amino acids 
C. Blood organic acids 
D. Urine amino acids 
E. Urine organic acids 
 
10.3. Other initial metabolic tests to consider would be: 
A. Blood glucose 
B. Blood carnitine 
C. Blood amino acids 
D. Blood ammonia 
E. All of the above 
 
10.4. The disease category that best describes this child is:  
a. Catastrophic disease of the newborn 
b. Sudden infant death syndrome (SIDS)  
c. Large molecule storage disease 
d. Urea cycle disorder 
e. Amino acid disorder 
 
10.5. The finding most suggestive of metabolic disease is:  
A. A prior child dying from infection 
B. Abnormal blood pH 
C. Increased amount of jaundice 
D. Consanguinity in the parents 
E. Absence of documented infection in the baby 
 
10.6-10.10.  
A  2-year-old  child  presents  with  lethargy  and  decreased  appetite.  On  physical  examination,  she 
appears  dehydrated  with  an  enlarged  liver.  Initial  testing  reveals  a  low  blood  glucose  of  55 
milligrams  per  deciliter,  a  pH  of  7.3,  and  a  slightly  low  bicarbonate  of  15  milliequivalents  per  liter. 
The prior childhood and family history are normal. A urine test was run and was negative. 
 
10.6. The urine test is most likely:  
A. Urine pH 
B. Urine reducing substances 
C. Urine organic acids 
D. Urine glucose 
E. Urine ketones 
 
10.7. The negative ketone test is suggestive of the category of: 
a. Amino acid disorders 
b. Carbohydrate disorder 
c. Nonketotic hypoglycemia 
d. Nonketotic hyperglycinemia 
e. Organic acidemias 
 
10.8. The most likely diagnosis in this child is: 
A. Organic acidemia 
B. Fatty acid oxidation disorder 
C. Urea cycle disorder 
D. Amino acid disorder 
E. Glycogen storage disease 
 
10.9. Additional tests that should be considered are: 
A. Urine organic acids, urine reducing substances 
B. Blood amino acids, blood carnitine 
C. Urine organic acids, blood carnitine and acylcarnitine profile 
D. Blood ammonia, liver biopsy for enzyme assay 
E. Blood amino acids, urine reducing substances 
 
10.10.  After  the  preliminary  tests  were  completed,  a specific mutation was demonstrated using the 
polymerase  chain  reaction  and  allele-specific  oligonucleotide  hybridization.  The  most  likely 
diagnosis is: 
A. Maple syrup urine disease 
B. Galactosemia 
C. Long chain hydroxyacyl-CoA dehydrogenase (LCHAD) deficiency  
D. Short chain acyl-CoA dehydrogenase (SCAD) deficiency 
E. Medium chain acyl-CoA dehydrogenase (MCAD) deficiency 
 
 
 
 

2009 FINAL  
Ps: i’m really confused about the order of the questions.  
 
1) Mark the false sentence describing autosomal recessive inheritance 
a. The pattern of pedigree is vertica; 
b. If the disease is very rare- the parents are probably related 
c. Males and Females are at equal risk for being affected 
d. Carriers are not always healthy 
 
2) Mark the disease doesn’t caused by expanded triplet repeats 
a. Fragile X syndrome 
b. Duchenne muscular dystrophy 
c. Myotonic dystrophy  
d. Huntington disease 
 
3) Complete but age-dependent penetrance is characteristic for 
a. Sickle cell disease 
b. Myotonic dystrophy 
c. Huntington disease 
d. Achondroplasia 
 
4) Which disorders are examples of uniparental disomy 
a. Duchenne and Becker muscular dystrophy 
b. Prader-Willi and Angelman syndrome 
c. Alpha and beta thalassemia 
d. Diabetes Type I and II 
 
5) Mark the false sentence describing mitochondrial inheritance 
a) Is called maternal inheritance because affected females transmit the disease to offspring 
b) Neural and muscle cells are rich in mitochondria therefore neuropathies and myopathies are the 
most common mitochondrial diseases 
c) Mitochondrial DNA particles are circular 
d) Sperm cells have mitochondria in tail and head 
 
6) The karyotype 46, XY can occur in females with  
a. Testicular feminization syndrome 
b. Turner syndrome 
c. Triple X syndrome 
d. Triploidy  
 
7) Rett syndrome, lethal in hemizygous males, is an example of 
a. Dominant X-linked inheritance 
b. Recessive X-linked inheritance 
c. Co-dominant inheritance 
d. Autosomal dominant inheritance 
 
8) Duchenne muscular dystrophy-recessive X-linked disease can occur in female being a carrier 
a. With a new mutation on second X chromosome 
b. With Turner syndrome 
c. Showing atypical lyonization 
d. All answers are appropriate 
 
9) Mark the false sentence describing Fragile X syndrome 
a. Occurs both in males and females with identical frequency 
b. Is the most common cause of mental retardation in males 
c. Is recessive X-linked disease 
d. Can be detected using banding techniques 
 
10)... 
 
b. recessive X-linked inheritance 
c. Co-dominant inheritance 
d. Autosomal dominant inheritance 
 
11) Duchenne muscular dystrophy-recessive X-linked disease can occur in female being a carrier 
e. With a new mutation on second X chromosome 
f. With Turner syndrome 
g. Showing atypical lyonization 
h. All answers are appropriate 
 
12) The karyotype 46, XY can occur in females with  
e. Testicular feminization syndrome 
f. Turner syndrome 
g. Triple X syndrome 
h. Triploidy  
 
13) Mark the karyotype of a mosaic male half normal and half Philadelphia 
a. 46,XY / 46,XY, t(9;21) 
b. 46,XY / 46,XY t(9;22) 
c. 46,XY / 47,XY t(9;22) 
d. 46,XY / 46,XY t(14;21) 
 
14) Which statement (s) are correct 
1. G and Q banding are novel techniques which are likely to replace traditional FISH 
2. G banding is performed with Giemsa dye 
3. Chromosome painting allows differential visualization of all human chromosomes 
4. FISH technique can be used only in metaphase stage​
a. 1 and 2 
b. 2, 3 and 4 
c. 1 and 4 
d. 2 and 3 
 
15)  The  fact  that  known  chromosomal  autosomal  aneuploidies  in  the  live  newborns  are  limited  to 
crisomies of chromosomes 21, 13 or 18 is related to 
1. Resistance to other human chromosomes to meiotic nondisjunction  
2. Embryonic lethality of all the other autosomal aneuploidies in human 
3. Unavailability of methods allowing a large scale study of the prevalence of other aneuploidies 
4. Occurrence of multiple fragile sites on chromosomes 21, 13, or 18 
a. 1 and 4 
b. 3 only 
c. 2 only 
d. All 
 
12) The acrocentric chromosomes are 
a. 20, 21 and 22 
b. 13, 14 and 15 
c. 1, 2 and 3 
d. X and Y 
 
13) mark the false statement for chromosome X inactivation 
a. Inactivation occurs in somatic and germ line cells 
b. Occur in early embryonic life 
c. Is random 
d. Inactive chromosome is present as a Barr body  
 
14) Different mutations within dystrophin gene cause 
a. Galactosaemia 
b. Cystic Fibrosis 
c. Myotonic dystrophy 
d. Huntington disease 
 
15) The karyotype can be performed from 
a. Skin cells 
b. Amniotic fluid 
c. Blood cells 
d. All answers are correct 
 
16) Which statement about Patau syndrome is false 
a. The frequency increases dramatically in mothers over the age of 35 
b. The cause is a mitochondrial mutation 
c. Affected individuals have an extra copy of 13 autosome 
d. There are some cases of a robertsonian translocation 
 
17) DNA denaturation in PCR method requires 
a. Low temperature 
b. F 
c. F 
d. F 
 
1) Mark the false sentence describing X-linked recessive inheritance 
a. The pattern of pedigree is like a knight’s move 
b. The father never transmits the disorder to his sons 
c. Males and females are at equal risk for being affected 
d. Carriers are females 
 
2) Variable expressivity is characteristic for 
A. Turner syndrome  
B. Duchenne muscular dystrophy  
C. Skin albinism 
D. Marfan syndrome 
 
3) Mark the false sentence describing mitochondrial inheritance 
a. Males do not transmit mitochondrial disorders 
b. Muscle cells are rich in mitochondria therefore myopathies are common mitochondrial disease 
c. Mitochondrial DNA particles are linear 
d. Sperm cells have only a few mitochondria 
 
4) Mark the trait which is not an example of discontinuous multifactorial trait 
a. Height 
b. Schizophrenia 
c. Cleft lip 
d. Neural tube defect 
 
5) The karyotype 46, XY can occur in females with  
a. Testicular feminization syndrome 
b. Turner syndrome 
c. Triple X syndrome 
d. Triploidy  
 
6) Mark the karyotype of a mosaic male half normal and half cri du chat 
 
1) Mark the false sentence describing autosomal recessive inheritance 
e. The pattern of pedigree is vertica; 
f. If the disease is very rare- the parents are probably related 
g. Males and Females are at equal risk for being affected 
h. Carriers are not always healthy 
 
 
2) Mark the disease NOT caused by expanded triplet repeats 
a) Colour blindness 
b) Duchenne muscular dystrophy 
c) Myotonic dystrophy 
d) Phenylketonuria 
 
3) Which disorders are examples of uniparental disomy? 
a. Duchenne and Becker muscular dystrophy 
b. Prader-Willi and Angelman syndrome 
c. Alpha and beta thalassemia 
d. Diabetes type I and II 
 
4) Mark the false sentence describing mitochondrial inheritance 
a. Is called maternal inheritance because affected females transmit the disease to offspring 
b. Neural and muscle cells are rich in mitochondria therefore neuropathies and myopathies are the 
most common mitochondrial diseases 
c. Mitochondrial DNA particles are circular 
d. Sperm cells have mitochondria in tail and head 
 
5) Mark the trait which is not an example of continuous multifactorial trait 
a. Height 
b. Skin colour 
c. Cleft lip 
d. Intelligence 
 
6) Rett syndrome, lethal in hemizygous males, is an example of 
a. Dominant X-linked inheritance 
b. Is the most common cause of mental retardation in males 
c. Is recessive X-linked disease 
d. Can be detected using banding techniques 
 
7) The karyotype 46, XY can occur in females with  
a. Testicular feminization syndrome 
b. Turner syndrome 
c. Triple X syndrome 
d. Triploidy  
 
8)  The  fact  that  known  chromosomal  aneuploidies  in  live  newborns  are  limited  to  trisomies  of 
chromosomes 21, 13, or 18 is related to  
1. Resistance to other human chromosomes to meiotic nondisjunction  
2. Embryonic lethality of all the other autosomal aneuploidies in human 
3. Unavailability of methods allowing a large scale study of the prevalence of other aneuploidies 
4. Occurrence of multiple fragile sites on chromosomes 21, 13, or 18 
a. 1 and 4 
b. 3 only 
c. 2 only 
d. All 
 
9) The acrocentric chromosomes are 
a. 13,14,15,21 and 22 
b. 9,10 and 11 
c. 20,21, and 22 
d. X and Y 
 
10) 80% of achondroplasia cases are a result of  
a. Genomic imprinting 
b. New mutations 
c. MItochondrial mutations 
d. Pleiotropy  
 
11) Increased number of triplets within FMR1 gene cause 
a. Galactosemia 
b. Fragile X syndrome 
c. Alzheimer disease 
d. Huntington disease 
 
12) Cytogenetics is a study of  
a. Chromosomes and their abnormalities 
b. .. 
c. .. 
d. .. 
 
7) Becker muscular dystrophy - recessive X-linked disease can occur in a female being a carrier 
a. With a new mutation on second X chromosome 
b. With Turner syndrome 
c. Showing atypical lyonization 
d. All answers are appropriate 
 
8) The karyotype 46, XY can occur in females with  
e. Testicular feminization syndrome 
f. Turner syndrome 
g. Triple X syndrome 
h. Triploidy  
 
9) ) Mark the karyotype of a mosaic male half normal and half Philadelphia 
e. 46,XY / 46,XY, t(9;21) 
f. 46,XY / 46,XY t(9;22) 
g. 46,XY / 47,XY t(9;22) 
h. 46,XY / 46,XY t(14;21) 
 
10) Which statement (s) are correct 
5. G and Q banding are novel techniques which are likely to replace traditional FISH 
6. G banding is performed with Giemsa dye 
7. Chromosome painting allows differential visualization of all human chromosomes 
8. FISH technique can be used only in metaphase stage 
e. 1 and 2 
f. 2, 3 and 4 
g. 1 and 4 
h. 2 and 3 
 
19)  Sequence  GCAATATTCCGGTAATAGCACT  was  cut by enzyme recognizing GCA. How many bands will 
be seen in electrophoresis analysis 
a. One band 
b. Two bands 
c. Three bands 
d. Four bands 
 
20) Join the disorder with the inheritance type 
1. Myoclonic epilepsy a. Autosomal dominant 
2. Hyperthermia of anesthesia b. Mitochondrial 
3. Color blindness c. X-linked recessive 
 
…………………………………………………………………………………………………………… 
 
21.  In  a certain population Gaucher disease occurrence is 1 per 10 000 humans. Calculate the frequency 
of the carriers. 
 
 
22. The pedigree shows …​X LINKED RECESSIVE​….. Inheritance 
 
 
 
 
 

MCQ-2 
1. Which would NOT be an effective way to diagnose sickle cell anemia 
a) PCR exon 1 of the Beta-globin gene, followed by restriction digest with MstII  
b) Hemoglobin electrophoresis 
c) Restriction  digest  of  genomic  DNA  with  MstII,  followed  by  Southern  blot,  using  a  Beta-globin 
gene product 
d) Enzymatic assay for hemoglobin activity  
 
2. A human male carrying an allele for a trait on the X chromosome is : 
a) Heterozygous 
b) Homozygous 
c) Hemizygous  
d) Monozygous  
 
3. Mark the sentence that is false for chromosome X-inactivation: 
a) Inactivation occurs in somatic and germ line cells 
b) Occurs in early embryonic life 
c) Is random 
d) Inactive chromosome is present as a Barr body  
 
4.  Which  of  the  following  procedures  is  used  as  a  routine  technique  for  karyotyping  using  light 
microscopy? 
a) C-banding 
b) Fluorescence in situ hybridization (FISH) 
c) G-banding 
d) Q-banding 
 
5.  If  a  person  with  blood  type  AB  mates  with  someone  with  blood  type  0,  what  are  the possible blood 
types of theis offspring? 
a) AB, 0 
b) A,B,0 
c) A,B 
d) A,B,AB,0 
 
6. Hemolytic Disease of Newborn (HDN) is a result of: 
a) Rh factor conflict between Rh(-) mother and Rh(+)child 
b) Rh factor conflict between Rh(+)mother and Rh(-) child 
c) Being mother affected with sickle cell anemia  
d) Being mother infected with malaria during pregnancy 
 
7.  For  a  given  autosomal  recessive  disease,  q=0.02  (where  q  is  the  allele  frequency  of  the  mutant 
allele). Approximately what percentage of the population has two copies of the normal allele. 
a) 19% 
b) 1.9% 
c) 96% 
d) 98% 
 
8. Mark the false sentence describing mitochondrial inheritance: 
a) Only affected females transmit the disease to offspring 
b) Neuropathies and myopathies are the most common mitochondrial diseases 
c) Mitochondrial DNA particles are circular single stranded 
d) Different cells vary in number of mt DNA copies 
 
9. Late paternal age can result in higher frequency of: 
a) Genomic imprinting 
b) New mutations 
c) Mitochondrial mutations 
d) Chromosomal aberrations 
 
10. Mark the disease which is not caused by expanded triplet repeats: 
a) Myotonic dystrophy 
b) Duchenne muscular dystrophy 
c) Huntington disease 
d) Fragile X syndrome 
 
11. Rett syndrome, lethal in hemizygous males, is an example of: 
a) Dominant X-linked inheritance 
b) Recessive X-linked inheritance 
c) Co-dominant inheritance 
d) Mitochondrial inheritance 
 
12. Mark the false sentence describing Fragile X syndrome: 
a) Isone of the most common causes of mental retardation in males 
b) The frequency is 1 in about 2000 male live births ​
c) Can be detected using banding technique and molecular methods ​
d) Is dominant X-linked disease  
 
13. Which of the disorders, except Tay-Sachs disease, is also more common in Ashkenazi Jews: 
a) Gaucher disease 
b) Sickle cell anemia 
c) Cystic fibrosis 
d) Niemann-Pick disease 
 
14. Which of the following is NOT a chromosomal aberration? 
a) Frame shift 
b) Deletion  
c) Translocation 
d) Inversion 
 
15. Which disorders are examples of uniparental disomy: 
a) Prader-Willi and Angelman syndrome 
b) Duchenne and Becker muscular dystrophy 
c) Alpha and beta thalassemia 
d) Trisomy 21 and 22  
 
16. The karyotype 46,XX del Xp can occur in females with: 
a) Testicular feminization syndrome 
b) Turner syndrome 
c) “Superwoman” syndrome 
d) None of these 
 
17. Which statement about Down Syndrome is false? 
a) The frequency increases in mothers over the age of 35 
b) The cause is nondisjunction 
c) The only aberration is a simple trisomy of 21 pair 
d) There are patients with translocation i.e t(14;21) 
 
18. Match the karyotypes with the probable phenotypes 
1. Fragile X syndrome a.neonatal death 
2. 45,XX b. Reduced fertility 
3. Unbalanced translocations carrier c. mental retardation  
 
a) 1a, 2c, 3b 
b) 1c, 2a, 3b 
c) 1a, 2b, 3c 
d) 1b, 2a, 3c 
 
19. The acrocentric chromosomes are: 
a) 21, 22 and Y 
b) 13,14 and 15 
c) 13,14,15,21 and 22 
d) X and Y 
 
20. Karyotyping in prenatal diagnosis is important in cases of: 
a) Pregnancy of females who had miscarriages 
b) Pregnant females that already have 1 child with congenital defects 
c) c) pregnant females after 36 
d) All above 
 
21. The banding techniques are used for detection of: 
a) All structural aberrations 
b) Number and big structural aberrations 
c) Microdeletions  
d) Aneuploidies  
 
22. FISH as a technique for mutation diagnosis in human genetics is essential for detection of: 
a) Angelman syndrome  
b) Turner syndrome 
c) XYY males 
d) Cri du chat syndrome 
 
23.  Chronic  Myeloid  Leukemia  is  caused  by  translocation  between  ABL  and  BCR  genes  which 
determines male karyotype: 
a) 46,XY, t(9;21) 
b) 46,XY, t(9;22) 
c) 46,XY, t(21;22) 
d) 46,XY, t(11;22) 
 
24. Many different mutations within CFTR gene cause: 
a) Galactosemia  
b) Tay - Sachs disease 
c) Cystic fibrosis 
d) albinism 
 
25. Fluorescently labelled molecular probes are used in: 
a) PCR 
b) FISH 
c) AFLP 
d) SSCP 
 
26. PCR-RFLP can be useful in: 
a) Related species identification  
b) Point mutation detection  
c) Population studies 
d) All answers are correct 
 
27. VNTR sequences are mostly used in: 
a) Point mutations determination  
b) Detection of pathogens 
c) Forensic medicine 
d) Chromosome aberration study 
 
28. The frequency of phenylketonuria is 1 per 7000 (in Europe). What is the frequency of carriers? 
a) 1.195% 
b) 0.988% 
c) 2.361% 
d) 1.180% 
 
29.  All  of  the  following  karyotypes  are  found  in  spontaneous  abortuses.  Which  of  the  following  is 
least likely to be found in a live-born infant? 
a) 45,X 
b) 47,XX, +21 
c) 47,XX, +16 
d) 47,XY, +18 
 
30. Which of the following syndromes is associated with maternal disomy for chromosome 15? 
a) Prader Willi syndrome 
b) Angelman syndrome  
c) Klinefelter´s syndrome 
d) Rett Syndrome 
 
31. Fragile-X syndrome is associated with: 
a) Chromosome 16 abnormalities 
b) A chromosomal deletion at Xq28 
c) Expansion of a CAG triplet repeat 
d) Expansion of a CGG triplet repeat  
 
32.  A  nineteen  year  old  female  with  short  stature,  wide  spaced  nipples,  and  primary  amenorrhea 
most likely has the karyotype of: 
a) 47,XXX 
b) 47,XXY 
c) 45,X 
d) 47,XX, +18 
 
33. The majority of cases of Down syndrome occur as a result of: 
a) Nondisjunction during paternal meiosis 
b) End to end fusion of two chromosomes 21  
c) Nondisjunction during maternal meiosis 
d) Mosaicism of normal and trisomic cell lines 
 
34. A person with two or more different cell lines originating from a single zygote is called a: 
a) Hemizygote 
b) Chimaera 
c) Mosaic 
d) Carrier 
 
35. Technique useful for person identification is: 
a) DNA fingerprint analysis 
b) Northern blot 
c) SSCP analysis 
d) RFLP assay 
 
36. Nucleic acid amplification is performed in vitro by: 
a) PCR and Southern 
b) Southern and Northern 
c) PCR and RT-PCR 
d) RT-PCR and Northern 
 
37. It is NOT possible to detect frameshift mutation by: 
a) PCR and RT-PCR 
b) Western (immunological) techniques 
c) Southern techniques 
d) Northern techniques 
 
38. Molecular techniques are useful for: 
a) Point mutations detection 
b) gene(s) expression analysis 
c) Parasites detection and determination 
d) All mentioned above 
 
39. For eukaryotic gene(s) expression in Escherichia coli it is obligatory to use: 
a) cDNA and expression vector 
b) Genomic DNA and expression vector 
c) cDNA and YAC vector 
d) Genomic DNA and YAC vector 
 
40. Molecular assay for paternity determination can involve : 
a) Mitochondrial DNA polymorphism analysis 
b) Ribosomal DNA polymorphism analysis 
c) Microsatellite DNA polymorphism analysis 
d) Promoter polymorphism analysis 
 
41. What kind of inheritance is the pedigree showing? 

 
a) Mitochondrial  
b) X-linked recessive 
c) Autosomal dominant 
d) X-linked dominant 
 
42. The pedigree pattern is an example of: 
 

 
a) Autosomal dominant inheritence 
b) Autosomal recessive inheritance 
c) X-linked recessive inheritance 
d) X-linked dominant inheritance  
 
 

MUSCULAR DYSTROPHY QUIZ - during seminars.  


 
1. What is muscular dystrophy? 
a) Diseases that destroy the bodies muscles 
b) Disease that makes muscles unproportional 
c) Being born with a too much or too little muscles 
d) Disease that makes muscles not grow 
 
2.  Most  patients  with  Duchenne  muscular  dystrophy  exhibit  clinical  signs  of  muscle  weakness 
between: 
a) 0-2 years 
b) 2-5 years 
c) 5-7 years 
d) 7-9 years 
 
3.  The  pattern  of  weakness  that  predominates  in  the  early  stages  of Duchenne muscular dystrophy 
is: 
a) Proximal muscles and lower extremities  
b) Distal muscles and upper extremities 
c) Proximal muscles and upper extremities 
d) Distal muscles and lower extremities 
 
4. All of the following tests are helpful in diagnosing Duchenne muscular dystrophy except: 
a) Pelvic X-ray 
b) Electromyography 
c) Serum levels of muscle enzymes 
d) Muscle biopsy 
 
5. Duchenne muscular dystrophy is associated with a defect on chromosome 
a) 1 
b) 5 
c) 21 
d) X-chromosome 
 
6. All of the following regarding Becker muscular dystrophy are correct except 
a) Dystrophin is not produced at all 
b) Occurs due to deletions in dystrophin gene 
c) It is less severe than Duchenne muscular dystrophy 
d) Some functional dystrophin is produced 
e) Age of onset is 8-25 years 
 
7.  The  couple  has  son  with  Becker  muscular  dystrophy.  Genetic  test  have  shown  that  mother  is  a 
carrier  of  dystrophin  mutation,  however  she  does  not  exhibits  any  abnormalities  in  clinical 
examination  .  What  is  the  probability  that  the  next  child  of  this  couple  will  suffer  from  Becker 
muscular dystrophy.  
a) Each son has 50% chance of being affected, each daughters has a 50% chance of being a carrier 
b) each child, regardless of its sex, has 50% chance of being affected 
c) each child, regardless of its sex, has 25% chance of being affected 
d) because the couple already has the affected son, their next child will be unaffected. 
 
8. What role does dystrophin play in the muscle cell? 
a) Dystrophin works to increase the ability of actin and myosin to bind together 
b) Dystrophin  is  a  transport  protein  that  helps  control  the  intracellular  environment  of  the 
myocyte 
c) Dystrophin works to link the cytoskeleton with the extracellular matrix 
d) None of the above 
  
 
SPECIAL QUESTIONS    

Questions 1-3 

1.  A  term  female  infant  to  a  37-year-old  mother  with  three  prior  children has a low birth weight 
and  a  poor  latch  for  breastfeeding  the  first  24  hours  of  life.  Mother  had  first  trimester  maternal 
serum  screening  (quad  screen)  that  was  normal.  Your  assessment  of  the  baby  reveals  an  unusual 
facial  appearance  with  a  broad  nose  and extra skin folds on the neck. Based on the history, which of 
the following is the most likely reason for poor breastfeeding in this child:  
A. Maternal incompetence 
B. Autosomal dominant disorder in mother 
C. X-linked recessive disorder in child 
D. Chromosomal disorder in child 
E. Multifactorial disorder in child 

2.  Which  of  the  following  is  an  important  test  to consider for this infant along with the expected 
time to receive results? 
A. Routine blood karyotype, 2-4 hours 
B. Routine blood karyotype, 2-4 days 
C. DNA chip for Mendelian disorders, 2-3 days 
D. DNA chip for Mendelian disorders, 2-3 weeks 
E. Neonatal blood spot, 2-4 hours 
F. Neonatal blood spot, 7-10 days 
 

3.  Prior  to  receiving  test  results,  the  most  important  aspect  of  care  along  with  evaluating  the 
feeding problem is: 
A. Genetic counseling regarding recurrence risk 
B. Genetic counseling regarding prenatal diagnosis 
C. Supportive counseling for future mental retardation 
D. Supportive counseling for probable birth defects 
E. Supportive counseling explaining the management plan  

Questions 4-5 

A  21-year-old  female  was  referred  to  obstetric clinic from the emergency room after a diagnosis 


of  malnutrition  and  a  positive  pregnancy  test.  She  had  been  brought  in  by  the  police  for  vagrancy 
and  alcoholism,  exhibiting  poor  hygiene  and  nutrition  on  examination.  She  also  was  affected  with 
cystic  fibrosis,  having  a  milder  disease  course,  and  a  sister  had  a  child  with  spina  bifida.  Fetal 
ultrasound  revealed  a  fetus  of  about  3  months  gestation  with  very  small  head  circumference, 
abnormal head shape, and intrauterine growth retardation.  
 

4.  The  poor  nutrition  and  unplanned  pregnancy  caused  the  young  woman  to  miss  the  following 
standards of care: 
A. Amniocentesis because of higher risks for chromosome abnormalities and cystic fibrosis 
B. Triple/Quad screening with ultrasound to screen for fetal chromosome abnormalities 
C. Preconception counsel including provision of vitamins with folic acid 
D. Prosecution because of suspected alcoholism causing damage to the fetus 
E. Preimplantation genetic diagnosis of to avoid the high risk for fetal cystic fibrosis 
 

5. Which of the following birth defects would be most likely to occur in this situation? 
A. Congenital heart defect 
B. Omphalocele 
C. Anencephaly 
D. Tracheoesophageal fistula 
E. Anal atresia 
 

6.  A  Caucasian  couple  in  the  20s  comes  in  for  preconception  counseling  regarding  their  first 
pregnancy.  They  have  had  no  prior  miscarriages  or  infertility  and  their  family  histories  are  normal. 
This  lack  of  risk  factors  means  that  their  risk  for  fetal  abnormalities  in  this  pregnancy  is 
approximately: 
A. 50% 
B. 25% 
C. 10% 
D. 2-3% 
E. <1% 
 

7. Which of the following genetic screening tests should be considered for this couple? 
A. Alpha-thalassemia 
B. Beta-thalassemia 
C. Tay-Sachs disease 
D. Sickle cell anemia 
E. Cystic fibrosis 

Questions 8-9 
8.  A  couple  present  to  an  obstetric  nurse  for  counseling  because  they  have  had  three  early 
miscarriages  at  6-8  weeks  gestation.  Both  are  in  good  health  without chronic illnesses, and neither 
has  any  family  history  of  birth  defects  or  miscarriages.  Which  of  the  following  is  an  important 
contributor to miscarriages that can be tested in this couple? 
A. Autosomal dominant disorders 
B. Chromosomal disorders 
C. Multifactorial disorders 
D. Mitochondrial disorders 
E. X-linked recessive disorders 
 

9.  Which  of  the  following  results  is  most plausible for this couple, along with its likelihood given 


their history? 
 
A. Trisomy, 1% 
B. Trisomy, 10% 
C. Translocation, 2-3% 
D. Translocation 20-30% 
E. Turner syndrome, 10% 
 

10.  A  pregnant  couple  had  a  prior  child with multiple birth defects who died at another hospital. 


During  the  history,  the  parents  describe  some  of  the  laboratory  results  that  were  obtained.  Which 
of the following descriptions is most accurate? 
A. A normal karyotype was obtained, ruling out a chromosome disorder. 
B. Blood was frozen so that a chromosome study could be performed. 
C. Blood was obtained on us (the parents), ruling out a problem in our child 
D. Our son had a normal karyotype, so he cannot carry the gene for cystic fibrosis like our daughter 
does. 
E. Blood  was  obtained  for  chromosome  studies,  so  you  should  be  able  to  determine  our  blood 
types. 
 
11.  A  couple  who  both  have  Aa  genotypes  at  a  locus  will  produce  fertilized eggs in which of the 
following ratios?: 
A. 1AA : 1 Aa : 1 aa 
B. 1AA : 1 Aa : 1 aa 
C. 1AA : 2 Aa : 1 aa 
D. 1AA : 2 Aa : 2 aa 
E. 1AA : 3 Aa  
 
12.  A  couple  requests  counseling  to  determine  the  recurrence  risk  for  albinism (203100).  Their 
first  son  is  affected,  consistent  with  the  presence  of  two  abnormal  alleles  (genotype  aa).  No other 
family  members  are  affected.  The  mother  and  father  are  first  cousins  and  the  grandfathers  are 
brothers.  The  mother  has  four  older  sisters  and  the  father  is  an  only child. The couple’s recurrence 
risk for their next child to have albinism is: 
A. 100% 
B. 75% 
C. 50% 
D. 25% 
E. Less than 1% 
 

13.  A  man  is  affected  with  polydactyly,  an  autosomal  dominant  trait  that  produces  an  extra 
finger  on  the  ulnar  (little  finger)  side.  What  is  the  risk  that  the  man’s  first  two  children  will  have 
polydactyly? 
(A) 100% 
(B) 75% 
(C) 50% 
(D) 25% 
(E) Virtually 0 
 

14. All the following statements regarding autosomal dominant conditions are true except that: 
A. they produce a vertical pattern in pedigree 
B. they affect both sexes 
C. they transmit through both sexes 
D. they often exhibit variable expressivity 
E. their expression requires the presence of two abnormal alleles at a locus 
 

15-18. Match the descriptions below with the appropriate method of DNA analysis. 
(A) Southern blot detecting globin gene deletion 
(B) Southern blot detecting amplified DNA fragment  
(C) Polymerase chain reaction followed by hybridization to oligonucleotide probes 
(D) Hemoglobin electrophoresis 
(E) Northern blot demonstrating increased RNA transcription 
 

15. Diagnosis of thalassemia ​A 


16. Diagnosis of mutant alleles ​C 

17.  Diagnosis  of  diseases  with  unstable  triplet  repeats  like  fragile  X  syndrome  or  Huntington 
chorea ​B 

18. Diagnosis of altered protein ​D 


 

19-20. Match the following descriptions with the terms below: 


(A) A recombinant DNA molecule that contains a DNA sequence of interest 
(B) A DNA molecule into which the DNA sequence of interest is cloned 
(C) Radioactive or fluorescent labeled DNA or RNA fragment used for hybridization 
(D) A DNA sequence that enhances RNA transcription 
(E)  Oligonucleotide  designed  to  catalyze  DNA  amplification  in  sequencing  or  polymerase  chain 
reactions 

19. Probe 

20. Primer 
 

21.  A  couple  is  referred  to  the  physician  because  the  first  three  pregnancies  have  ended  in 
spontaneous  abortion.  Chromosomal  analysis  reveals  that  the  wife  has  two  cell  lines  in  her  blood, 
one  with  a  missing  X  chromosome  (45,X)  and  the  other  normal  (46,XX).  Her  chromosomal 
constitution can be described as 
(A) chimeric 
(B) monoploid 
(C) trisomic 
(D) mosaic 
(E) euploid 

Questions 22-23 

22.  A  couple  in  their  20s  present  to  an  obstetric  nurse  for  counseling  because  they  had an early 
miscarriage where studies showed disorganized fetal tissue and a karyotype of trisomy 16.  Both are 
in  good  health  without  chronic  illnesses,  and  neither  has  any  family  history  of  birth  defects  or 
miscarriages. What would be an important recommendation for their next pregnancy? 
A. Parental chromosomes 
B. First trimester quad screen 
C. Chorionic villus sampling 
D. Planned pregnancy and routine care 
E. Preimplantation genetic diagnosis 
 

23.  Given  appropriate testing, what is the risk for chromosome aberrations in the next pregnancy 


to this couple? 
A. 100% 
B. 50% 
C. 25% 
D. 20-30% 
E. <1% 
 

24.  A  child  with  severe  kidney  failure  has  what  chance  that  his  or  her sibling will have identical 
haplotypes at the HLA C locus? 
A. 100% 
B. 75% 
C. 50% 
D. 25% 
E. virtually 0% 

25. Which of the following parent-offspring units would be at highest risk for Rh disease? 
A. mother C+, father D+, baby D+, first pregnancy 
B. mother E+, father D+, baby E+, second pregnancy 
C. mother D+, father CD+, baby CD+, second pregnancy 
D. mother DC+, father D+, baby CD+, second pregnancy   
E. mother CE+, father D+, baby CDE+, second pregnancy 
 

26.  A  physician  wishing  to  prevent  ABO  incompatibility  from  occurring  in  his  type  O  daughter’s 
pregnancies should recommend: 
A. Denial of type A or type B suitors 
B. Administration of blocking antibodies to A antigen if she has a pregnancy with a type A fetus. 
C. Denial of type O suitors 
D. Administration of blocking antibodies to B antigen if she has a pregnancy with a type B fetus 
E. Free selection of suitors because ABO incompatibility does not occur 
 

27.  What  is  the  risk  for  any  type  of  fetal  incompatibility  to  a  type  A,  Rh  C+ mother and a type B, 
Rh  D+  father  who  are  beginning  their  second  pregnancy?  It  is known from the grandparental blood 
types that these parents are heterozygous at all blood group loci. 
A. 100% 
B. 75% 
C. 50% 
D. 25% 
E. Virtually 0 
 

28-33. Match the following risk factors with the syndrome categories below: 
A. a chromosomal syndrome 
B. a deformation syndrome 
C. a Mendelian syndrome 
D. a disruption syndrome 
E. a malformation syndrome 
 
28. Oligohydramnios (scanty amniotic fluid) 

29. Renal agenesis 

30. Advanced paternal age 

31. Advanced maternal age 

32. Multiple miscarriages 

33. Tearing of the amnion 


 

34-36.  Match  each  of  the  ethnic  groups  listed  below  with  the  genetic  disorder  commonly 
associated with it. 
(A) Cystic fibrosis 
(B) α-thalassemia 
(C) Tay-Sachs disease 
(D) α-1-antitrypsin deficiency 
(E) Glucose-6-phosphate dehydrogenase deficiency 

34. African Americans 

35. Chinese Americans 

36. Jewish Americans 


 

37-41. The various types of genetic anemias, like many other genetic disease categories, must be 
diagnosed  by  particular  laboratory  studies.  Match  the  following  alterations  of  ​β​-globin  gene 
expression with the test that will be sufficiently abnormal to allow diagnosis. 
(A). Mutation in the upstream region to cause mild thalassemia 
(B). Deletion of the second β-globin exon to cause severe thalassemia 
(C). Mutation in an RNA splice site to cause severe thalassemia 
(D). Amino acid substitution (charged to neutral) to produce an unstable β-globin peptide 
(E). Amino acid substitution (neutral to neutral) in the binding site for heme. 

37. Analysis to detect altered mobility of ​β​-globin protein by electrophoresis  

38. Analysis to detect decreased oxygen binding by red blood cells  

39. Analysis to detect altered size of ​β​-globin protein 

40. Analysis to detect altered DNA sequence of promoter regions 

41. Analysis to detect altered size of ​β​-globin mRNA 


 

42. The cytogenetic term “6q+” refers to 


(A) 46,XX,dup(6q) 
(B) extra chromosome material derived from the long arm of chromosome 6 
(C) 46,XX,dup(6p) 
(D) extra chromosome material, origin unspecified, attached to the long arm of chromosome 6 
(E) 47,XX,+6 

43–45. Match each clinical situation below with the appropriate risk figure. 
(A) 1/10,000 
(B) 1/800 
(C) 1/100 
(D) 1/10 
(E) 1 

43. The risk for a newborn to have Down syndrome 

44. The theoretical risk for a 21/21 translocation carrier to have a child with Down syndrome 

45.  The  risk  for  parents  of  a  trisomy  21  child  to  have  a  second  offspring  with  a  chromosomal 
abnormality 
 

46–48. Match each of the genetic conditions below with the correct cytogenetic notation. 
(A) 47,XX,+21 
(B) 45,X 
(C) 47,XXX 
(D) 47,XY,+21 
(E) 45,XX,-21 

46. Male with trisomy 21 (Down syndrome) 

47. Female with monosomy X (Turner syndrome) 

48. Female with monosomy 21 


 

49.  Monozygotic  twins  with  connected  placental  circulations  can  develop  a  pattern  of  vascular 
occlusions  due  to  blood  clots.  A twin with a brain cyst, absent kidney, cleft palate, and absent digits 
has a: 
A. Malformation syndrome 
B. Deformation syndrome 
C. Disruption syndrome 
D. Dysplasia syndrome 
E. Malformation sequence 
 

50.  At  her  first  obstetric  visit,  a  woman  tells  you  she  has  a  brother  with  mental  disability.  She 
asks what the risk for mental disability will be for her current pregnancy. You reply: 
A. Mental disability is a complex phenotype that is rarely genetic. 
B. Mental disability fits into the polygenic category with a low recurrence risk. 
C. It is imperative to establish a more specific diagnosis before counseling can be provided. 
D. It is imperative to perform a karyotype on her brother before counseling can be provided. 
E. The risk is significant but there is no prenatal diagnosis for mental disability.   
   

51. An individual with genotype Aa at a genetic locus will produce: 


A. Only gametes with genotype Aa 
B. Only gametes with genotype a 
C. Only gametes with genotype aa 
D. Half of gametes with genotype A and half with genotype a 
E. Only gametes with genotype AA 

52. A  woman  has  two  brothers  with  mental  disability,  and  her  mother  also  had  a  brother  with 
mental  disability  plus  two  with  normal  cognitive  function.  The  woman’s  would  occur  if  her 
fetus was: 
A. Male—50% 
B. Male—25% 
C. Male <1% 
D. Male or female—50% 
E. Male or female—25% 

  

53.  If  allele  B  causes  disease  and  allele  b  is  associated  with  a  normal  phenotype,  what  is  the 
chance  that  a  baby  born  to  a  Bb  mother  will  have  the  disease?  Assume  that  the  father  has  a  bb 
genotype and that there is no variable expressivity. 
A. 100% 
B. 75% 
C. 50% 
D. 25% 
E. Less than 1% 
  

  54.  The  allele  for  normal  hemoglobin  is represented as A, and that for sickle hemoglobin as S.  A 


man  with  sickle  cell  trait  (genotype  AS)  marries  a  woman  who  is  also  sickle  trait.  What  are  their 
chances to have a child with sickle cell anemia (genotype SS)?  
A. 100% 
B. 75% 
C. 50% 
D. 25% 
E. Less than 1% 
 

55.  A  woman is distraught because she has had a child with spina bifida and believes her episode 
of influenza at 6 months gestation is responsible. You reply: 
A. Influenza does not cause birth defects. 
B. The  neural  tube  forms  in  the  first  month,  so  infections  in  the  second  trimester  should  not 
affect it. 
C. Congenital infections always produce syndromes, not single birth defects. 
D. Spina bifida is due to an underlying Mendelian disorder 
E. Spina bifida is due to an underlying chromosome disorder   
 

56-58.  Many  of  the  more  common  birth  defects  like  cleft  palate  or  congenital  heart  disease 
exhibit  multifactorial  determination. Although specific empiric risks can be specified as in Tables 4.2 
and  4.3,  general  risks  can  be  borne  in  mind  relative  to  an  affected  person:  identical  twin,  20-30%; 
first-degree  relative,  3-4%;  two  first-degree  relatives,  5-8%;  three  first-degree  relatives,  9-12%; 
second-degree  relatives,  0.7-2  %,  third-degree  relatives  and  general  population,  less  than  0.5%.  By 
reference  to  the  person with a birth defect, match the relatives below with their proportion of genes 
in common and their concordance or recurrence risk: 
(A) 100% genes in common, 20-30% concordance risk 
(B) 50% genes in common, 3-4% concordance risk 
(C) 50% genes in common, 3-4% recurrence risk 
(D) 25% genes in common, 2% recurrence risk 
(E) 12.5% genes in common, <0.5% recurrence risk 

56. Twin brother whose twin sister has cleft palate by ultrasound 

57. Unborn sibling of a child with congenital heart defect 

58. Grandchild of a person with spina bifida 


 

59-60.  For  each  clinical  presentation,  discuss  risks  for  the  next  pregnancy and the best prenatal 
diagnosis option.   

59.  A  couple  in  their  early  30’s  has  a  child  with  the  trisomy  21  form  of  Down  syndrome;  they 
want the earliest and most accurate prenatal diagnosis.. 
A. Risk for Down syndrome 1%--first trimester quad screen plus ultrasound 
B. Risk for Down syndrome 1%--chorionic villus sampling 
C. Risk for Down syndrome 1%--amniocentesis 
D. Risk for Down syndrome 10%--first trimester quad screen plus ultrasound 
E. Risk for Down syndrome 10%--chorionic villus sampling 

60.  A  couple  in  their  20’s  has  a  child  with  spina  bifida.  They  want  the  safest method of prenatal 
diagnosis.  
A. Risk for spina bifida 2%--first trimester quad screen plus ultrasound 
B. Risk for spina bifida 2%--chorionic villus sampling 
C. Risk for spina bifida 2%--amniocentesis 
D. Risk for spina bifida 10%--first trimester quad screen plus ultrasound 
E. Risk for spina bifida 10%--amniocentesis 
 

Вам также может понравиться